Test Bank – ACP: Oncology

Cancer Epidemiology and Prevention

1. A 25-year-old white woman presents to your clinic for a routine examination. She is feeling well, but she is unhappy about the fact that she has gained 10 lb since graduating from college. She attributes the weight gain to her new investment banking job. Her job is highly stressful, and she reports that her company’s office culture revolves around “happy hour.” She denies using tobacco, but she frequents smoky bars. On physical examination, the patient appears well nourished; her body mass index is 25. The examination is otherwise unremarkable.

Which of the following statements regarding primary cancer prevention is true?
A. In nonsmokers, long-term passive exposure to tobacco smoke is associated with a significantly increased risk of lung cancer
B. A diet that includes antioxidant supplements, such as ß-carotene, reduces the risk of lung cancer
C. Stress reduction has been proved to decrease the incidence of gastric cancer associated with gastric ulcers
D. Annual Papanicolaou smears have reduced the incidence of adenocarcinoma of the vagina

Key Concept/Objective: To understand the factors that increase the risk of cancer

Long-term exposure to environmental tobacco smoke (passive smoking) has been associated with a 30% increase in the risk of lung cancer in nonsmokers. ß-Carotene has been associated with an increase in the risk of lung cancer. Helicobacter pylori, not stress, is the causal agent in gastric cancer. Papanicolaou smears are used for the secondary prevention of cervical cancer. (Answer: A—In nonsmokers, long-term passive exposure to tobacco smoke is associated with a significantly increased risk of lung cancer)

2. During a routine office visit, a 49-year-old woman of Ashkenazi Jewish descent expresses concern about her risk of cancer. She reports that her younger sister has just been diagnosed with breast cancer. In addition, the patient tells you that her father was recently found to have an adenomatous polyp on colonoscopy. Her mother died in an automobile accident at an early age; she was otherwise healthy.

Which of the following statements about factors that predispose to cancer is true?
A. The retinoblastoma gene (Rb-1) is inherited in an autosomal recessive pattern
B. Familial colon cancer has been linked to germline mutations in DNA
repair genes such as MSH2, MLH1, MSH6, PMS1, and PMS2
C. Hereditary breast cancer resulting from mutations in the BRCA1 and BRCA2 genes account for the majority of all breast cancers in Ashkenazi Jews
D. Major susceptibility loci for hereditary prostate cancer has been mapped to the Y chromosome

Key Concept/Objective: To understand that genetic alterations underlie the transformation of a normal cell to a cancerous cell

Approximately one third of retinoblastomas occur in an autosomal dominant pattern with high penetrance. In contrast, familial colon cancer without multiple polyposis may be caused by germline mutations in one of the DNA repair genes: MSH2, MLH1, MSH6, PMS1, or PMS2. BRCA1 and BRCA2 account for most of the hereditary breast cancers in young women; carriers of BRCA1 are also predisposed to ovarian cancer of early onset. Hereditary prostate cancer, which accounts for 5% to 10% of all cases, is primarily associated with disease of early onset. Major susceptibility loci for hereditary prostate cancer were recently mapped to chromosome 1 and the X chromosome. (Answer: B—Familial colon cancer has been linked to germline mutations in DNA repair genes such as MSH2, MLH1, MSH6, PMS1, and PMS2)

For more information, see Neugut AI, Li FP: 12 Oncology: I Cancer Epidemiology and Prevention. ACP Medicine Online (www.acpmedicine.com). Dale DC, Federman DD, Eds. WebMD Inc., New York, November 2004

Molecular Genetics of Cancer

3. A 51-year-old male patient recently presented with splenomegaly and weight loss. He was diagnosed as having chronic myelogenous leukemia (CML). He has done some reading on his own and is inquisitive about the etiology of this cancer.

Which of the following statements regarding the molecular genetics of CML is false?
A. The chromosomal translocation in CML involves the c-myc gene
B. Philadelphia chromosome is associated with CML
C. The accelerated or blast phase of CML is often associated with duplication of the Philadelphia chromosome
D. Imatinib mesylate is directed against the tyrosine kinase produced by the Philadelphia chromosome and is therefore used in the treatment of CML

Key Concept/Objective: To understand the molecular genetics of CML

The first specific chromosomal translocation identified in human cancer was the Philadelphia chromosome, which underlies CML. The fusion of chromosomes 9 and 22 leads to the joining of two unrelated genes, the c-abl gene, which encodes a tyrosine kinase and is located on chromosome 9, and the gene bcr (for breakpoint recombination), located on chromosome 22. A chimeric protein with novel transforming properties is formed from this specific chromosomal rearrangement. The accelerated or blast phase of CML is often associated with duplication of the Philadelphia chromosome, suggesting that increased copies of this aberrant gene confer a dose-dependent transforming effect. The recent discovery of an effective inhibitor of the bcr-abl kinase, imatinib mesylate (formerly STI571), has led to dramatic responses in CML and has revolutionized treatment of this leukemia. In Burkitt lymphoma, the chromosomal locus containing the c-myc gene is rearranged such that the upstream negative regulatory regions (i.e., regions located to the 5′ side of a gene) of c-myc are lost; expression of the gene is directed by the strong immunoglobulin heavy-chain enhancer, which is constitutively active in B cells. Deregulation of c-myc expression in these cells is thus a potent force driving cellular proliferation. (Answer: A—The chromosomal translocation in CML involves the c-myc gene)

4. A 31-year-old female patient comes to your office with concerns about her family being “predisposed” to cancer. She says that her son was recently diagnosed with retinoblastoma. She wants to know if future children will also develop cancer.

Which of the following statements regarding the Knudson model of human cancer genetics is false?
A. Children with familial tumors have inherited an initial genetic hit (i.e., a distinct genetic alteration) and require only one additional, rate-limiting genetic hit to initiate tumorigenesis
B. Tumor suppressor gene mutations are gain-of-function mutations; they are dominant mutations
C. Both retinoblastoma and Wilms tumor follow the Knudson model of human cancer genetics
D. Children with sporadic tumors need to acquire two independent genetic hits within the same cell

Key Concept/Objective: To understand the Knudson model of human cancer genetics

Alfred Knudson proposed the model that now forms the foundation of human cancer genetics. The Knudson model predicts that children with familial tumors have inherited an initial genetic hit (i.e., a distinct genetic alteration) and require only one additional, rate-limiting genetic hit to initiate tumorigenesis. In contrast, children with sporadic tumors need to acquire two independent genetic hits within the same cell, an unlikely event that explains the less frequent, unilateral presentation and later onset of sporadic cancers. Subsequent genetic studies in two of the tumors studied by Knudson identified these so-called genetic hits as the sequential inactivation of the two alleles of a critical tumor suppressor gene: RB1 in retinoblastoma and WT1 in Wilms tumor. The Knudson model also explains the paradox that tumor suppressor gene mutations are loss-of-function or recessive mutations, yet familial cancer presents as an autosomal dominant trait. Although loss of a single allele of a tumor suppressor gene may be functionally silent in the presence of a normal second allele, the frequency of spontaneous mutations is sufficiently high to ensure that at least one cell within the target tissue is likely to lose the second allele and initiate malignant transformation. (Answer: B—Tumor suppressor gene mutations are gain-of-function mutations; they are dominant mutations)

5. A 57-year-old man presents to the hospital with generalized weakness, weight loss, and worsening constipation. He is found to be profoundly anemic. His stools test positive for occult blood. A colonoscopy is performed, and a large intraluminal mass is seen. Results of biopsy indicate adenocarcinoma of the colon.

Which of the following statements regarding p53 is false?
A. p53 plays a critical role in the maintenance of genomic integrity and is known as the “guardian of the genome”
B. Genetic injuries trigger the stabilization and activation of p53 protein
C. p53 protein functions by repairing DNA molecules
D. High levels of p53 protein in tumor specimens are commonly taken as evidence of a mutation in p53

Key Concept/Objective: To understand the basic principles of the p53 genomic stability gene

p53 plays a critical role in the maintenance of genomic integrity—hence its popular designation as “guardian of the genome.” The p53 protein is normally expressed at low levels in all cells. However, genetic injuries, such as those that occur through ionizing radiation, trigger the stabilization and activation of p53 protein. p53 functions as a transcription factor, directing expression of p21, an inhibitor of the cyclin-dependent kinases that regulate the cell cycle. Activation of p53 leads to arrest in the G1 phase of the cell cycle, enabling cells to repair DNA damage before proceeding into S phase and DNA replication. In other cells, activation of p53 causes activation of multiple effectors, leading to apoptosis—a suicide program in cells whose DNA may have been irreparably damaged. Not surprisingly, mutations of p53 are common in human cancers, being demonstrable in about 50% of cases. Most mutations are amino acid substitutions within the DNA-binding domain of p53, resulting in its misfolding and binding to heat shock proteins. The rate of protein turnover is greatly slowed for these mutant p53 molecules. This explains the paradox that high levels of p53 protein in tumor specimens are commonly taken as evidence of a mutation in p53. (Answer: C—p53 protein functions by repairing DNA molecules)

6. A 75-year-old woman presents with left flank pain, weight loss, and bilateral lower extremity edema. She is noted to have microscopic hematuria and erythrocytosis on laboratory evaluation. A CT scan of her abdomen and pelvis reveals a large left renal cell carcinoma with near total occlusion of the inferior vena cava by the tumor. It is known that certain tumor suppressor genes can undergo mutation and lead to cancer.

Which of the following statements regarding tumor suppressor genes is false?
A. von Hippel-Lindau gene (VHL) is frequently mutated in adult renal cell cancers
B. The WT1 gene encodes a transcription regulator that is specifically expressed in podocytes of the developing glomerulus; mutations in WT1 cause Wilms tumor, an embryonic kidney cancer
C. SMAD genes, active in signaling by transforming growth factor–a (TGF-a), are mutated in pancreatic tumors
D. The APC gene is a key target in breast cancer

Key Concept/Objective: To know the tumor suppressor gene mutations that are involved in common solid tumors

The identification of tumor suppressor genes implicated in cancer predisposition syndromes led to the discovery of key components of cellular differentiation pathways. The WT1 gene encodes a transcription regulator that is specifically expressed in podocytes of the developing glomerulus. Mutations in WT1 cause Wilms tumor, an embryonic kidney cancer. VHL is frequently mutated in adult renal cell cancers and in the germline of persons who have a syndrome that includes both benign and malignant vascular tumors. The VHL protein appears to be involved in the regulation of protein degradation pathways, particularly that of the transcription factor hypoxia-inducible factor. The APC gene is a key target in colorectal cancer: germline mutations cause familial polyposis coli, a syndrome characterized by the development of numerous colonic polyps that are at very high risk for malignant transformation; somatic mutations constitute the earliest step in the development of colorectal cancer. The SMAD genes, active in signaling by TGF-a, are mutated in pancreatic tumors. (Answer: D—The APC gene is a key target in breast cancer)

For more information, see Haber DA: 12 Oncology: II Molecular Genetics of Cancer. ACP Medicine Online (www.acpmedicine.com). Dale DC, Federman DD, Eds. WebMD Inc., New York, June 2004

Principles of Cancer Treatment

7. A 74-year-old woman presents to your office complaining of a breast mass. On physical examination, the patient is found to have a 2 × 3 cm mass in the right breast. A mammogram is obtained, and the results are suspicious for cancer. A fine-needle aspiration biopsy shows adenocarcinoma. As part of your evaluation before deciding the best therapy to offer, you wish to determine the stage of her malignancy.

What is the usual method for staging the majority of tumors?
A. Tumor size, presence of comorbidities, and the involvement of lymph nodes
B. Tumor size, involvement of lymph nodes, and the presence of metastasis
C. Histology type, tumor size, the involvement of lymph nodes, and metastasis
D. Performance status, involvement of lymph nodes, and metastasis

Key Concept/Objective: To understand tumor staging

Knowledge of the extent of the disease at the time of diagnosis (stage) is required to properly manage patients with cancer. Staging is based on three components: the size or depth of penetration of the tumor (T), the involvement of lymph nodes (N), and the presence or absence of metastases (M). The TNM staging system is now standard; its use is required not only for the management of cancer cases but also for the reporting of cancer cases to cooperative groups and many tumor registries. Within each TNM category, the extent of involvement correlates with prognosis and can help in deciding the appropriate treatment. Although there are few exceptions, the majority of tumors are staged on the basis of this classification. Although the performance status and the presence of comorbidities are important factors that should be considered when deciding the best therapy for the patient, they are not part of the staging system. (Answer: B—Tumor size, involvement of lymph nodes, and the presence of metastasis)

8. A 37-year-old man is diagnosed with Hodgkin lymphoma after presenting with fever and lymphadenopathy. The patient is treated with a combination of doxorubicin, bleomycin, vincristine, and dacarbazine. After 3 days of treatment, the patient complains of mouth pain and diarrhea. His physical examination shows erythematous buccal mucosa.

Which of the following is the most likely pathogenesis of these complications?
A. Progression of his lymphoma
B. Vitamin B12 deficiency
C. Clostridium difficile colitis
D. A side effect of the chemotherapy

Key Concept/Objective: To understand the most common mechanisms of toxicity of antineoplastic drugs

Most antineoplastic drugs target proteins or nucleic acids that are common to both malignant and nonmalignant cells and thus have a narrow therapeutic index. In addition, antineoplastic drugs are usually administered at very high doses. Several toxicities are shared by many antineoplastic drugs. These include nausea and vomiting, mucositis and diarrhea, myelosuppression, alopecia, infertility, and increased risk of secondary malignancy. Nausea and vomiting result from local gastrointestinal effects as well as activation of the chemoreceptor trigger zone in the central nervous system. Mucositis and diarrhea are attributable to the relatively high proliferative rate of normal gastrointestinal tissues, which makes these tissues more susceptible to the cytotoxic effects of many chemotherapeutic regimens. Similar mechanisms result in chemotherapy-induced myelosuppression, alopecia, and infertility. Many chemotherapeutic agents are found to be mutagenic when tested in vitro. Moreover, epidemiologic studies of patients cured of a pediatric cancer have demonstrated that certain therapies are associated with a higher risk of secondary malignancies. (Answer: D—A side effect of the chemotherapy)

9. A 33-year-old woman is diagnosed with non-Hodgkin lymphoma. She is treated with chemotherapy and has a good initial response; however, after a few courses of therapy, she develops progressive disease. The oncologist considers drug resistance as the cause of this therapeutic failure and mentions the presence of P-glycoprotein as a possible mechanism.

Of the following, which best describes the mechanism of resistance associated with P-glycoprotein?
A. Induction of mutations of the receptor to which the antineoplastic drug binds
B. Induction of angiogenesis
C. Recognition of the antineoplastic drug and pumping of the drug to the extracellular space
D. Induction of hepatic microsomes and an increase in the metabolic rates of different antineoplastic agents

Key Concept/Objective: To understand the mechanisms of resistance to antineoplastic drugs

For an anticancer drug to kill a cancer cell, the drug must enter the bloodstream, be activated or escape inactivation by drug-metabolizing enzymes, and reach the target in its active form. The drug-drug-target interaction must then produce cell death. Resistance to anticancer therapy results from interference with one or more of these critical steps. Some of the factors affecting these steps are poor absorption or distribution of the drug; metabolism of the drug in the liver; decreased blood supply to the target organ; hypoxia; changes in the receptors; and mechanisms that block the intracellular accumulation of the drug. Multidrug resistance refers to the clinical and laboratory circumstance in which a tumor is no longer susceptible to several chemotherapeutic drugs having different mechanisms or targets. P-glycoprotein is a member of the adenosine triphosphate (ATP)-binding cassette family. P-glycoprotein spans the plasma membrane and recognizes a broad spectrum of anticancer drugs. In the presence of ATP, P-glycoprotein pumps the drugs to the extracellular space, so that effective concentration at the intracellular target is never achieved. (Answer: C—Recognition of the antineoplastic drug and pumping of the drug to the extracellular space)

10. A 58-year-old man presents with hemoptysis and a weight loss of 5 kg over the past month. Chest x-ray shows a 1 cm nodule in the right upper lung, near the periphery. Biopsy confirms non–small cell lung cancer. Node sampling is negative, and no metastases are found. The patient smokes one pack of cigarettes a day but is willing to quit. His medical history includes chronic obstructive pulmonary disease, hypertension (well controlled with medications), and stable angina with a normal electrocardiogram. His FEV1 is 700 ml.

Which of the following is the determining factor in the decision regarding whether or not to pursue curative therapy in this patient?
A. The size of the tumor
B. The patient’s node-negative status
C. The patient’s stable angina
D. The fact that the patient smokes
E. The patient’s FEV1 value

Key Concept/Objective: To understand the rationale for palliative therapy versus curative therapy

This patient has a potentially curable tumor. It is small and distant from the mediastinum, and he has no positive lymph nodes or metastases (T1N0M0). Although he still smokes, he is willing to consider quitting, and smoking is not an absolute contraindication to curative surgery. His stable angina is also not a contraindication to surgery. However, his FEV1 of less than 800 ml is below the lower limit for safe resection of the tumor. He would not have enough reserve lung capacity to survive the surgery. For this reason, therapy should be aimed at palliation. In general, the extent of involvement described by the TNM system correlates with prognosis. In addition to cancer prognosis, tolerability of therapy is crucial to decisions regarding palliation or goal of cure because aggressive therapy may be harmful to patients, as in this case. (Answer: E—The patient’s FEV1 value)

11. The patient in Question 10 wants your advice about the treatment options for his cancer. Clinical trials relevant to his condition are available. He is very well educated and wants to know which ones he should participate in. You tell him that there are several phase III trials available, and he asks what a phase III trial studies.

Which of the following is investigated in a phase III trial?

A. The maximum tolerated dose of an antineoplastic agent
B. Efficacy of treatment as measured by changes in the size of tumors and time to progression
C. Efficacy of treatment as measured by survival rates and quality of life
D. Efficacy of a new active agent compared with that of the best available therapy

Key Concept/Objective: To understand the phases of treatment trials

Phase I clinical trials identify the maximum tolerated dose of a new drug. Phase II clinical trials assess efficacy by use of change in tumor size, quality of life, disease-progression parameters, and survival. Phase III clinical trials compare a new chemotherapeutic agent with the best available therapy. (Answer: D—Efficacy of a new active agent compared with that of the best available therapy)

12. A 43-year-old woman presents with a 2 cm breast mass. Excision biopsy and node dissection reveal an aggressive carcinoma with 6 of 10 axillary nodes positive. Combination chemotherapy is recommended. The patient is concerned about combination therapy and wishes to have single-agent therapy.

You explain that combination chemotherapy is desirable because it does which of the following?
A. Reduces the risk of secondary malignancy
B. Increases cure rates by decreasing the risk of cross-resistance
C. Decreases resistance-conferring mutations by allowing larger doses of each agent to be given
E. Eliminates the need for radiation therapy
F. Decreases the risk of gastrointestinal side effects

Key Concept/Objective: To understand the rationale for combination chemotherapy

The Goldie-Coldman model predicts drug resistance by use of cell number and the spontaneous cancer cell mutation rate. Even a tumor that is too small to be detected clinically has a significant chance of containing a cell with a resistance-conferring mutation. Although combination chemotherapy allows for reduction of the dosage of any one agent, it does not inherently reduce side effects or the risk of mutation-induced secondary malignancy or eliminate the need for radiation therapy (which reduces the risk of local recurrence). Finally, combination therapy does not allow for higher dosages. If anything, combination chemotherapy necessitates lower dosages of agents that have common toxicities. Combination chemotherapy attempts to address possible cross-resistance by employing different mechanisms, and nonoverlapping toxicities allow for effective dosing. (Answer: B—Increases cure rates by decreasing the risk of cross-resistance)

13. A 55-year-old man returns to the office 2 months into treatment for metastatic prostate cancer. His treatment includes prostatectomy, nilutamide, and radiation. He now reports tender, enlarged breasts, whitish nipple discharge, nausea, and diarrhea. He has no ill contacts and has recently returned from a trip to Arizona. He also reports that he has stopped drinking alcohol because it makes him feel ill. Examination results are as follows: temperature is 99.5° F (37.5° C); pulse, 88 beats/min; and blood pressure, 120/82 mm Hg. Breast examination confirms gynecomastia and galactorrhea. Abdominal examination shows mild tenderness in the lower abdomen without rebound. Rectal examination shows no masses, and the stool is heme-negative.

Which of the following is the best step to take next for this patient?
A. Determine serum prolactin and testosterone levels
B. Order a mammogram
C. Send stool sample to assess for enteric pathogens and ova and parasite
D. Reassure the patient that these are common side effects of his chemotherapy
E. Arrange for urgent head CT with contrast

Key Concept/Objective: To know the common side effects of antiandrogenic chemotherapy

The common side effects of antiandrogenic chemotherapeutic agents such as nilutamide include nausea, diarrhea, and constipation. Hormonal effects include gynecomastia, galactorrhea, breast tenderness, hot flashes, and decreased facial hair. Idiosyncratic reactions associated with nilutamide include delayed dark-light adaptation, interstitial pneumonitis, and alcohol intolerance. Thus, in this case, the patient needs to be reassured that these are usual side effects of his regimen. (Answer: D—Reassure the patient that these are common side effects of his chemotherapy)

For more information, see Rubin EH, Hait WN: 12 Oncology: IV Principles of Cancer Treatment. ACP Medicine Online (www.acpmedicine.com). Dale DC, Federman DD, Eds. WebMD Inc., New York, May 2003

Colorectal Cancer

14. A 55-year-old white man presents to your primary care clinic for his annual physical examination. He underwent a colonoscopy 2 years ago, and an adenomatous polyp was removed. After the examination, the patient mentions that he has been reading about colon cancer and polyps in the news and wants to know his risk of having colon cancer.

Which of the following statements regarding the relationship between colon cancer and polyps is false?
A. Most colorectal cancers arise from preexisting adenomas
B. Adenomatous polyps, as well as juvenile polyps, hamartomas, and inflammatory polyps, progress to colorectal carcinoma
C. Larger polyps, especially those larger than 1 cm, are more likely to contain invasive carcinoma
D. On the basis of histology, villous polyps are more likely to contain invasive carcinoma than are tubular polyps
E. Fewer than 1% of adenomatous polyps become malignant

Key Concept/Objective: To understand the relationship between various types of polyps and colorectal cancer

It is thought that most colorectal cancers arise from preexisting adenomas. Such potentially premalignant lesions should be distinguished from juvenile polyps, hamartomas, and inflammatory polyps, which are not thought to progress to colorectal cancer. Histologically, adenomatous polyps may be tubular, villous, or both (tubulovillous). The larger the adenoma, the greater the likelihood that a villous component will be present. Villous polyps are more likely to contain invasive carcinoma than are tubular polyps of the same size. Regardless of histologic class, large polyps—especially those larger than 1 cm in diameter—are more likely to contain invasive carcinoma. Fewer than 1% of adenomatous polyps ever become malignant. (Answer: B—Adenomatous polyps, as well as juvenile polyps, hamartomas, and inflammatory polyps, progress to colorectal carcinoma)

15. A 45-year-old woman presents to your office to establish primary care. While taking her medical history, you notice she has a strong family history of colon cancer occurring at a young age. You suspect hereditary nonpolyposis colorectal cancer (HNPCC).

Which of the following is NOT a part of the Amsterdam-2 criteria for identifying patients with HNPCC?

A. Histologically documented colorectal cancer (or other HNPCC-related tumor) in at least three relatives, one of whom is a first-degree relative of the other two
B. Cases of colorectal cancer in at least two successive generations of the family
C. A family history of one or more cases of colorectal cancer diagnosed before 60 years of age
D. Affected relatives must be on the same side of the family (maternal or paternal)

Key Concept/Objective: To know the diagnostic criteria for HNPCC

HNPCC is an autosomal dominant disorder associated with an unusually high frequency of cancers in the proximal large bowel. The median age at which adenocarcinomas appear in HNPCC is less than 50 years, which is 10 to 15 years younger than the median age at which they appear in the general population. Also, families with HNPCC often include persons with multiple primary cancers; in women, an association between colorectal cancer and either endometrial or ovarian carcinoma is especially prominent. Several sets of selection criteria have been developed for identifying patients with this syndrome. The Amsterdam-2 criteria comprise the following: histologically documented colorectal cancer (or other HNPCC-related tumor) in at least three relatives, one of whom is a first-degree relative of the other two; a family history of one or more cases of colorectal cancer diagnosed before 50 years of age; and cases of colorectal cancer in at least two successive generations of the family. Affected relatives should be on the same side of the family (maternal or paternal), familial adenomatous polyposis (FAP) must be excluded in colorectal cancer cases, and tumors must be pathologically verified. (Answer: C—A family history of one or more cases of colorectal cancer diagnosed before 60 years of age)

16. A 50-year-old black male patient returns to your office for follow-up for hypertension. His hypertension is well controlled with hydrochlorothiazide and an angiotensin-converting enzyme inhibitor. Because the patient is 50 years old, you talk about colorectal cancer screening measures.

Which of the following statements regarding colorectal cancer screening is false?
A. A fecal occult blood test (FOBT) is equally useful at detecting adenomas and early-stage cancers
B. A case-control study demonstrated a risk reduction of 70% for death from cancers within reach of the sigmoidoscope
C. Colonoscopic polypectomy lowers the incidence of colorectal cancers by 50% to 90%, and the American Cancer Society currently recommends colonoscopy every 10 years, starting at age 50, for asymptomatic adults at average risk for colorectal cancer
D. There has not been a formal trial of double-contrast barium enema (DCBE) as a screening test for colorectal neoplasia in a general population

Key Concept/Objective: To understand colorectal cancer screening tests

Screening and early detection (secondary prevention) are important in influencing the outcome in patients with colorectal neoplasia. Many deaths from colorectal cancers could probably be averted by appropriate use of screening. The rationale for screening for colorectal neoplasia is twofold: First, detection of adenomas and their removal will prevent subsequent development of colorectal cancer. Second, detection of localized, superficial tumors in asymptomatic individuals will increase the surgical cure rate. The rationale for screening for the presence of blood in the stool is that large adenomas and most cancers bleed intermittently. Annual testing may allow detection of disease that, although undetected on previous occasions, has not yet reached an advanced and perhaps incurable stage. Compared with endoscopic tests, FOBT detects relatively few adenomas; the principal benefit of an FOBT program is to increase detection of early-stage cancers. A case-control study demonstrated a risk reduction of 70% for death from cancers within reach of the sigmoidoscope; the data suggested that the benefit may last as long as 10 years. The effectiveness of colonoscopy has been demonstrated by several studies. Observational, case-control, and prospective, randomized trials have shown that colonoscopic polypectomy lowers the incidence of colorectal cancers by 50% to 90%. The American Cancer Society currently recommends colonoscopy every 10 years, starting at age 50, for asymptomatic adults at average risk for colorectal cancer. Repeat examinations at more frequent intervals are indicated for patients at increased or high risk. There has not been a formal trial of DCBE as a screening test for colorectal neoplasia in a general population. A comparison study in patients who have undergone colonoscopic polypectomy found colonoscopy to be a more effective method of surveillance than DCBE. (Answer: A—A fecal occult blood test [FOBT] is equally useful at detecting adenomas and early-stage cancers)

17. A 62-year-old black male patient is in the hospital for evaluation of anemia with associated fatigue and weight loss. He was found to be heme-positive on rectal examination, and a colonoscopy was performed. A mass was found in his ascending colon; biopsy revealed adenocarcinoma. A CT scan of his chest, abdomen, and pelvis revealed a 3 cm mass in his ascending colon; there were no liver lesions or other metastatic disease and no intraperitoneal lymphadenopathy. The patient was taken to surgery for resection. Surgical pathology revealed invasive adenocarcinoma extending into the serosa, but no lymph node involvement.

Which of the following statements regarding this patient’s staging and prognosis is false?
A. The patient has stage B disease because no lymph nodes are involved and no distant metastasis was found
B. The fact that the patient’s primary tumor was larger than 2 cm is a poor prognostic factor
C. Because most recurrences after resection occur within 3 to 4 years, the cure rate is reasonably estimated by 5-year survival rates
D. Postoperatively, the carcinoembryonic antigen (CEA) level may serve as a measure of the completeness of tumor resection

Key Concept/Objective: To understand the staging and prognosis of colorectal carcinoma

The prognosis for patients with adenocarcinoma of the colorectum is closely associated with the depth of tumor penetration into the bowel wall and the presence or absence of regional lymph node involvement and distant metastases. The Dukes system has been applied to the TNM classification method, in which T represents the depth of tumor penetration; N, the presence or absence of lymph node involvement; and M, the presence or absence of distant metastases. Stage A (T1N0M0) cancers are superficial lesions that do not penetrate the muscularis and do not involve regional lymph nodes. Stage B cancers penetrate more deeply into the bowel wall without lymph node involvement. Stage C cancers involve regional nodes. Stage D cancers have metastasized to liver, lung, bone, or other anatomically distant sites. Because most recurrences after resection occur within 3 to 4 years, the cure rate is reasonably estimated by 5-year survival rates. Although CEA is an imperfect tumor marker, it can provide useful information for the management of colorectal cancer patients if its limitations and attributes are understood. Postoperatively, the CEA level may serve as a measure of the completeness of tumor resection. If a preoperatively elevated CEA value does not fall to normal levels within 4 weeks (a period that is twice the plasma half-life of CEA) after surgery, the resection was probably incomplete or occult metastases are present. In contrast to the prognosis for patients with most other solid tumors, the prognosis for patients with colorectal cancer is not influenced by the size of the primary lesion when corrected for nodal involvement and histologic differentiation. (Answer: B—The fact that the patient’s primary tumor was larger than 2 cm is a poor prognostic factor)

For more information, see Levin B: 12 Oncology: V Colorectal Cancer. ACP Medicine Online (www.acpmedicine.com). Dale DC, Federman DD, Eds. WebMD Inc., New York, February 2004

Pancreatic, Gastric, and Other Gastrointestinal Cancers

18. A 61-year-old black man presents to your clinic with a 6-month history of progressive esophageal dysphagia and weight loss. He has a history of hypertension and severe gastroesophageal reflux disease (GERD). He is admitted to the hospital, and an esophagogastroduodenoscopy (EGD) with biopsy is performed. The findings indicate a diagnosis of adenocarcinoma of the esophagus.

For this patient, which of the following statements regarding esophageal cancer is false?
A. Proton pump inhibitors have been shown to stop the progression of
Barrett esophagus to adenocarcinoma
B. In the United States, the incidence of adenocarcinoma is increasing and the incidence of squamous cell carcinoma (SCC) is decreasing
C. Barrett esophagus is a complication of chronic reflux disease and is associated with an increased risk of adenocarcinoma of the esophagus
D. Staging of esophageal cancer can involve CT scanning of the chest, abdomen, and pelvis; endoscopic ultrasound (EUS); and PET scanning
E. The mainstay of therapy for esophageal cancer is surgery

Key Concept/Objectives: To understand the risk factors, diagnosis, and treatment of esophageal adenocarcinoma

Esophageal cancer, which includes SCC and adenocarcinoma, is the ninth most common cancer worldwide. The annual rate of SCC of the esophagus per 100,000 population is declining, and the incidence of esophageal adenocarcinoma is rapidly increasing in the United States and other countries. GERD is a risk factor for esophageal adenocarcinoma. Barrett esophagus, a metaplastic change of the lining of the esophagus in which the normal squamous cell epithelium is replaced by columnar intestinal-type epithelium, is a complication of chronic reflux disease. It is associated with an increased risk of adenocarcinoma of the esophagus. Because GERD is a risk factor for esophageal adenocarcinoma and because Barrett esophagus is highly associated with the disease, there is increased clinical interest in pharmacologic, surgical, or endoscopic therapy to decrease the risk, as well as prevent the development, of adenocarcinoma of the esophagus. However, there is no evidence to suggest that proton pump inhibitors either stop the progression of Barrett esophagus to adenocarcinoma or lead to regression in the presence of metaplastic tissue. Once a diagnosis has been established and careful physical examination and routine blood tests have been performed, a CT scan of the chest, abdomen, and pelvis should be obtained to assess tumor extent, nodal involvement, and metastatic disease. However, CT scanning may underestimate the depth of tumor invasion and periesophageal lymph node involvement in up to 50% of cases. EUS has the advantage of being able to image distinct wall layers, thereby providing a representation of the depth of tumor invasion with an accuracy of up to 90% and detecting regional lymph node involvement with an accuracy of 75%. EUS also can detect local tumor recurrence at an early stage. EUS should be considered as a mandatory procedure for staging workup, especially for patients who are being considered for preoperative treatments. PET scanning has become widely available and may be an important tool for staging, with both a sensitivity and a specificity of approximately 90%. PET scanning is considered to be superior to CT scanning in the evaluation of distant metastases. Treatment options for esophageal cancer are based on the stage of the disease at presentation. Surgery remains the mainstay of treatment of esophageal cancer. It can be curative in persons with resectable local and locoregional disease. (Answer: A—Proton pump inhibitors have been shown to stop the progression of Barrett esophagus to adenocarcinoma)

19. A 68-year-old white man presents for evaluation of early satiety, mild epigastric pain, nausea, and weight loss. On examination, the patient is pale and thin. He has a palpable periumbilical lymph node. Rectal examination shows heme-positive brown stool. EGD reveals a large mass in the stomach, and biopsy is performed.

For this patient, which of the following statements regarding risk factors for gastric cancer is false?
A. Helicobacter pylori infection is a primary risk factor for gastric cancer
B. Hereditary syndromes such as HNPCC, FAP, the Li-Fraumeni syndrome, and the Peutz-Jeghers syndrome are associated with gastric cancer
C. Approximately 50% of gastric cancers involve familial clustering
D. Salted, smoked, and dried foods that contain high concentrations of nitrates may be associated with the development of gastric cancer

Key Concept/Objective: To understand the risk factors for gastric cancer

Gastric cancer is an aggressive neoplasm that has a marked variation in both incidence and mortality between different populations. A high incidence of gastric cancer is observed in Asia, South and Central America (Chile and Costa Rica), Eastern Europe, and the Middle East. Extensive research has identified factors and events that influence the initiation, promotion, and progression of stomach cancer. Gastric cancer is a disease of complex etiology involving multiple risk factors, including dietary, infectious, occupational, genetic, and preneoplastic factors. Salted, smoked, and dried foods contain high concentrations of nitrates, which are converted into carcinogenic nitrosamines and nitrites by anaerobic bacteria; diets rich in such foods may be associated with the development of gastric cancer. Infection with Helicobacter pylori, a gram-negative spiral bacterium, is a primary risk factor for gastric cancer. The first strong data came from three separate, nested case-control studies. The statistically significant relative risk ranged from 2.8 in a British population to
6.0 in a cohort of Japanese males living in Hawaii. H. pylori gastritis causes cell proliferation with increased risk of DNA damage, leading to inadequate repair and malignant transformation. Although most cases of gastric cancer appear to be sporadic, approximately 10% of cases involve familial clustering. Other hereditary cancer syndromes in which gastric cancer may occur include HNPCC, FAP, the Li-Fraumeni syndrome, and the Peutz-Jeghers syndrome. (Answer: C—Approximately 50% of gastric cancers involve familial clustering)

20. A 63-year-old black man presents to the emergency department with abdominal pain, dizziness, and nausea. He reports that he has lost about 20 lb during the past month and a half and that his abdominal pain has just recently become severe. It is midepigastric, gnawing, and radiates to his back. Over the past 3 days, he has had polyuria, but nausea has prevented him from being able to stay hydrated. On examination, the patient is cachectic, has dry mucous membranes, and is orthostatic. His abdomen is tender in the midepigastric region, and there is no palpable mass. Results of laboratory studies are notable for an elevated glucose level of 630 mg/dl and mild renal insufficiency; pancreatic enzyme levels are normal. You diagnose the patient as having diabetes mellitus of new onset, but you are concerned that he may have an underlying pancreatic malignancy.

For this patient, which of the following statements regarding pancreatic cancer is false?
A. Pancreatic cancer is more common in males than in females and is more common in blacks than whites
B. Tumor size is a very important predictor of resectability, with tumors larger than 4 cm having less than a 10% chance of being resectable and nonmetastatic
C. EUS is the single most accurate test for imaging and staging pancreatic carcinoma
D. Risk factors for pancreatic cancer include increasing age, tobacco smoking, chronic pancreatitis, and coffee ingestion
E. Surgical resection is the only curative modality for pancreatic cancer

Key Concept/Objectives: To understand the risk factors and initial workup for pancreatic cancer

Pancreatic cancer is the fourth leading cause of death from cancer in both males and females in the United States. Ninety-five percent of malignant pancreatic tumors are exocrine pancreatic cancers, two thirds of which occur in the pancreatic head and one third in the pancreatic body and tail; the remaining 5% of malignant lesions are mostly islet cell tumors. The incidence of pancreatic cancer is higher in males than in females and is higher in blacks than in whites. Tobacco smoking has been the most consistently demonstrated risk factor, implicated as a cause in roughly 30% of cases of pancreatic cancer. Age is also an extremely important determinant of risk. With increasing age, the risk of pancreatic cancer increases exponentially. Coffee and alcohol consumption do not seem to increase the risk of pancreatic cancer. Initial symptoms experienced by pancreatic cancer patients are insidious and relatively nonspecific (e.g., weight loss, anorexia, abdominal discomfort or pain, and nausea); this may delay the diagnosis for several months. Pain can be a presenting symptom and is usually associated with localized invasion of peripancreatic structures (e.g., splanchnic plexus and retroperitoneum), particularly from lesions located in the body or tail of the pancreas. Pain is typically described as gnawing and severe, radiating to the back and worsening in the supine position. The early diagnosis of a potentially resectable pancreatic cancer is extremely difficult because of nonspecific initial symptoms and poor sensitivity of noninvasive techniques such as CT and ultrasonography. EUS is the single most accurate test for imaging and staging pancreatic carcinoma and can clearly evaluate pancreatic mucosal, vascular, ductal, and parenchymal abnormalities, as well as lymph node metastases. Patients with clinical symptoms that may represent pancreatic cancer should have an initial standard CT scan or an abdominal ultrasound. If a pancreatic mass is suspected on one of these initial tests, further evaluation is necessary. If the tumor appears to be larger than 4 cm or appears unresectable, spiral CT with intravenous contrast and endoscopic retrograde cholangiopancreatography with fine-needle aspiration should be considered. On the basis of size alone, masses greater than
4 cm have less than a 10% chance of being resectable and nonmetastatic. Because surgical resection is the only curative modality for pancreatic cancer and because only 10% to 15% of patients present with resectable disease, the diagnosis, stage, and management are based on resectability. (Answer: D—Risk factors for pancreatic cancer include increasing age, tobacco smoking, chronic pancreatitis, and coffee ingestion)

21. A 46-year-old white man with a history of I.V. drug abuse and cirrhosis complicated by ascites presents with massive hematemesis. He is intubated in the emergency department and is treated with fluid resuscitation. Emergent EGD reveals esophageal varices, and band ligation is performed. The patient has never had portal hypertension before, so a workup is performed. It reveals a 4 cm mass in the liver. The serum a-fetoprotein level is checked and is found to be 440 ng/ml.

For this patient, which of the following statements regarding risk factors for hepatocellular carcinoma (HCC) is true?
A. Cirrhosis induced by hepatitis B virus (HBV) or hepatitis C virus (HCV), but not by alcoholism, is a risk factor for HCC
B. The recent increase in incidence of HCC in the United States is most likely attributable to the increasing rates of HBV infection
C. Hereditary hemochromatosis is not a risk factor for the development of HCC
D. The presence of hepatitis infection and concomitant heavy alcohol consumption are synergistic in the development of HCC

Key Concept/Objective: To understand the risk factors for HCC

HCC is the most common primary malignant tumor of the liver. It is the fifth most common malignancy in the world (564,000 cases a year) and the third-highest cause of cancerrelated deaths worldwide. HCC is most often a complication of liver cirrhosis caused by chronic infection by HBV, HCV, or alcohol. The incidence of HCC in the United States has increased from 1.4 per 100,000 population for the period from 1976 through 1980 to 2.4 per
100,000 for the period from 1991 through 1995. This increase is considered to be primarily related to an increase in HCV infection. Hereditary hemochromatosis is also a risk factor for the development of HCC. Diabetes mellitus may be associated with HCC in patients with chronic HCV infection, and a significant synergy exists between heavy alcohol consumption, hepatitis virus infection (both HBV and HCV), and diabetes mellitus and the development of HCC. Patients with other metabolic disorders or conditions that may lead to cirrhosis (e.g., a1-antitrypsin deficiency, type I glycogen storage disease, tyrosinemia, and even
biliary atresia) are also at risk for developing HCC. Other risk factors include long-time
ingestion of food contaminated with aflatoxins, metabolites of the mold Aspergillus flavus, exposure to oral contraceptives, and exogenous androgens. (Answer: D—The presence of hepatitis infection and concomitant heavy alcohol consumption are synergistic in the development of HCC)

For more information, see Sun W, Haller D: 12 Oncology: VI Pancreatic, Gastric, and Other Gastrointestinal Cancers. ACP Medicine Online (www.acpmedicine.com). Dale DC, Federman DD, Eds. WebMD Inc., New York, February 2004

Breast Cancer

22. A 40-year-old African-American woman presents to your clinic for an annual health examination. She has hypertension and hyperlipidemia, for which she takes hydrochlorothiazide and a statin, respectively. Otherwise, she is in good health, and she exercises regularly. Her main concern today is her risk of breast cancer. She has a close friend who was recently diagnosed with breast cancer, and she is now very worried that she might one day get it.

Which of the following statements regarding the risk factors for breast cancer is true?
A. Because of germline mutations of either BRCA1 or BRCA2, the breastovarian cancer syndrome is inherited in an autosomal recessive fashion
B. Reproductive risk factors include late menarche, early menopause, and increasing parity
C. The diagnosis of breast cancer in first-degree relatives younger than 50 years is associated with a threefold to fourfold increased risk
D. Women between the ages of 40 and 50 years are at greatest risk; 75% of all breast cancers are diagnosed in that age group

Key Concept/Objective: To understand the risk factors for breast cancer

In first-degree relatives younger than 50 years, the diagnosis of breast cancer is associated with a threefold to fourfold increased risk. Several familial breast cancer syndromes and their associated molecular abnormalities have been identified. These include the breastovarian cancer syndrome, which is attributed to germline mutations in either of two breast cancer susceptibility genes, BRCA1 and BRCA2. These mutations are inherited in an autosomal dominant fashion and can therefore be transmitted through both the maternal and the paternal lines. Reproductive risk factors include early menarche, late menopause, late first pregnancy, and nulliparity. All are felt to lead to a condition of prolonged estrogen exposure to the breast. (Answer: C—The diagnosis of breast cancer in first-degree relatives younger than 50 years is associated with a threefold to fourfold increased risk)

23. On clinical examination, a 54-year-old woman is noted to have a nontender mass in the upper outer quadrant of her left breast. There are no overlying skin changes, and there is no palpable adenopathy in the axilla. The other breast is without masses. A mammogram obtained 1 year ago was normal. You immediately set up an appointment for mammography, but your patient is obviously disturbed. She has several questions regarding the therapy for breast cancer.

Which of the following statements regarding breast cancer therapy is true?
A. For women with stage I or II breast cancer, the survival rate with breast conservation therapy involving lumpectomy and radiotherapy is identical to the survival rate with modified radical mastectomy
B. Sentinel lymph node mapping is difficult to perform and offers no benefit to axillary lymph node dissection
C. The benefit of tamoxifen is limited to 5 years, and therefore, the recommendation is to discontinue therapy after 5 years
D. Aromatase inhibitors offer a viable alternative to tamoxifen therapy for premenopausal women

Key Concept/Objective: To understand the basic principles of breast cancer therapy

Breast conservation therapy that involves lumpectomy with radiotherapy and modified radical mastectomy that involves removal of the breast and axillary nodes provide identical survival rates for women with stage I or II breast cancer. Sentinel lymph node mapping involves injection of a radioactive tracer, vital blue dye, or both into the area around the primary breast tumor. The injected substance tracks rapidly to the dominant axillary lymph node—the so-called sentinel lymph node. This node can be located by use of a small axillary incision and visual inspection or by use of a handheld counter. If the sentinel node is tumor free, the remaining lymph nodes are likely to be tumor free as well, and further axillary surgery can be avoided. The benefit of tamoxifen increases with the duration of treatment; the proportional reductions in 10-year recurrence and mortality were 47% and 26%, respectively, with 5-year regimens of tamoxifen therapy. The aromatase inhibitors specifically inhibit this conversion, leading to further estrogen deprivation in older women. Randomized trials have shown that the aromatase inhibitors (e.g., anastrozole, letrozole, and exemestane) provide efficacy similar or superior to that of tamoxifen, along with an acceptable side-effect profile for postmenopausal women with metastatic breast cancer. Given their mechanism of action, aromatase inhibitors should not be used for treatment in premenopausal women. (Answer: A—For women with stage I or II breast cancer, the survival rate with breast conservation therapy involving lumpectomy and radiotherapy is identical to the survival rate with modified radical mastectomy)

24. A 42-year-old woman presents for a routine health maintenance visit. She underwent menarche at age
13 and is still menstruating. She has never been pregnant. There is no history of breast cancer in her family. Several of her friends have recently been diagnosed with breast cancer, and she is concerned about developing it herself. She performs monthly breast self-examinations and has noted no abnormalities. On physical examination, her breasts are normal.

Which of the following statements regarding breast cancer screening is true?
A. Yearly mammography improves survival
B. Mammography will detect more than 95% of breast cancers
C. The combination of clinical breast examination and mammography improves survival
D. Mammography is recommended by several professional organizations but has not been shown to improve survival
E. Screening for the BRCA gene mutations is recommended

Key Concept/Objective: To understand the recommended modalities in breast cancer screening

Screening modalities used for breast cancer include breast self-examination, clinical breast examination, and mammography. Breast self-examination is recommended by the American Cancer Society and other organizations despite the failure of a large clinical trial to show any benefit of self-examination over observation. The combination of clinical breast examinations and screening mammography in women 50 to 69 years of age has been shown to prolong survival; this approach resulted in a 25% to 30% decrease in mortality and is recommended by numerous advisory panels. In women 40 to 50 years of age who are at average risk, there is considerable controversy about the proper screening strategy because there has been no convincing evidence of survival benefit with clinical breast examinations and mammography. The clinical breast examination is an important part of screening because mammography does not detect 10% to 15% of breast cancers. Screening for the BRCA1 and BRCA2 mutations, which are seen in some families with a strong history of breast cancer, has not been rigorously investigated. In patients with no significant family history, this test would not be advisable. (Answer: D—Mammography is recommended by several professional organizations but has not been shown to improve survival)

25. A 59-year-old woman comes to your clinic wanting to know if there is anything she can do to decrease her risk of breast cancer. Two of her four sisters developed breast cancer while they were in their 50s. She experienced menarche at 12 years of age and menopause at 55 years of age. She had one child at 34 years of age. She underwent two breast biopsies for suspicious masses, which revealed normal breast tissue. Since she was 50 years of age, she has undergone yearly screening mammography, the results of which have been normal. Her breast examination reveals no masses, and there is no axillary lymphadenopathy.

Which of the following statements is false?
A. Bilateral prophylactic mastectomy would reduce this patient’s breast cancer risk by approximately 90%
B. Treatment with daily tamoxifen for 5 years would reduce this patient’s breast cancer risk by approximately 50%
C. Lifestyle modifications, such as adherence to a low-fat diet, weight loss for obese patients, and smoking cessation, have been shown to reduce breast cancer risk
D. Tamoxifen therapy is associated with an increased incidence of endometrial cancer and pulmonary embolism
E. A clinical history that includes multiple benign breast biopsy results increases this patient’s breast cancer risk

Key Concept/Objective: To understand the risk factors and the primary prevention strategies for breast cancer

Risk factors for the development of breast cancer include age, early menarche, late menopause, older age at first live birth or nulliparity, number of breast biopsies, number of first-degree relatives with breast cancer, and biopsies showing atypical hyperplasia. Women at high risk who received a 5-year course of tamoxifen were found to have 50% fewer diagnoses of breast cancer compared with women at comparable risk who did not receive tamoxifen. This therapy is associated with an increased risk of endometrial cancer and pulmonary embolism. Evidence from case series indicate that bilateral prophylactic mastectomy is associated with a greater than 90% reduction in the incidence of breast cancer. Prophylactic surgery has not been compared with aggressive screening in combination with appropriate management of breast cancer and should be considered only for highrisk patients. Lifestyle modifications, such as adherence to a low-fat diet, weight loss for obese patients, and smoking cessation, have not been shown in prospective clinical trials to reduce breast cancer risk but are associated with decreased breast cancer risk in epidemiologic studies and are therefore recommended by some advisory groups. (Answer: C— Lifestyle modifications, such as adherence to a low-fat diet, weight loss for obese patients, and smoking cessation, have been shown to reduce breast cancer risk)

26. A 67-year-old woman returns to your clinic for follow-up. Two years ago, she was diagnosed with stage I breast cancer and treated with lumpectomy and radiation therapy. The tumor, 0.8 cm in its largest dimension, tested negative for estrogen and progesterone receptors. The patient’s oncologist has asked you to conduct surveillance visits. The patient has no complaints. She denies having fevers, chills, weight loss, shortness of breath, or pain. She has been performing monthly breast self-examinations and has noted no changes or masses. On physical examination, she has a well-healed scar on her left breast with no other abnormalities. There are no breast masses palpable, and there is no axillary or supraclavicular lymphadenopathy. Bilateral mammograms are normal.

Which, if any, additional recommended surveillance studies should be conducted?
A. No additional studies are needed
B. Complete blood count and liver function tests
C. Complete blood count, liver function tests, and chest radiographs
D. Complete blood count, liver function tests, chest radiographs, and tumor markers (CEA and CA 27.29)
E. Bone scan and tumor markers (CEA and CA 27.29)

Key Concept/Objective: To understand the follow-up surveillance of patients with early-stage breast cancer

Breast cancer is both a common and curable malignancy; increasingly, primary care physicians are being called upon to conduct follow-up surveillance for patients who have undergone treatment of breast cancer. The issue of which methods are appropriate for follow-up surveillance has been investigated in two randomized clinical trials. Both studies found that the standardized use of laboratory and diagnostic tests did not enhance survival or quality of life when compared with the use of tests chosen on the basis of individual patients’ symptoms and clinical examination results. Recommended follow-up surveillance measures are patient education regarding symptoms of recurrence; regular historytaking and physical examinations; monthly breast self-examinations; annual mammography; and age-appropriate screening for other cancers. Routine use of complete blood counts, chemistry panels, tumor markers, chest x-rays, CT, or bone scans is not recommended. (Answer: A—No additional studies are needed)

27. A 54-year-old woman presents with a breast mass she discovered 2 days ago. She has been performing breast self-examinations monthly and noted a new mass in the upper outer quadrant of her left breast. She denies having any breast pain or nipple discharge. She has experienced no weight loss, headache, shortness of breath, or bony pain. On examination, she is found to have a 0.5 cm × 1 cm hard, mobile mass that is easily palpated. No skin abnormalities or other masses are detectable by palpation in either breast. There is no axillary or supraclavicular lymphadenopathy, and the remainder of a detailed physical examination is normal. A biopsy is performed of the patient’s breast mass, which is found to have infiltrating ductal carcinoma. Results of a complete blood count, liver function tests, and metabolic panel are all within normal limits.

At this point, the interventions that would provide the best survival and least morbidity for this patient include which of the following?
A. Modified radical mastectomy
B. Lumpectomy with axillary lymph node dissection
C. Radiation therapy
D. Modified radical mastectomy and radiation therapy
E. Lumpectomy with axillary lymph node dissection and radiation therapy

Key Concept/Objective: To understand the local treatment of early-stage breast cancer

The local management of early breast cancer has changed significantly in recent years, as breast-conservation therapy (BCT) has been shown to have survival rates identical to those of more extensive surgeries, such as radical mastectomy and modified radical mastectomy (MRM). In multiple clinical trials, a combination of lumpectomy and radiation therapy has yielded survival rates equivalent to those of MRM. Radiation therapy is a critical component of BCT because it reduces the recurrence rate from 40% to less than 10%. Axillary node dissection is important in diagnosis because positive nodes confer a worse prognosis and would prompt systemic chemotherapy. Sentinel node biopsy, in which the dominant axillary node is sampled and examined for tumor, is currently under investigation. If the sampled sentinel node is negative for tumor, the patient is spared the axillary node dissection and its morbidity. In general, BCT is preferred to MRM; MRM is indicated in cases in which radiation is contraindicated (such as in patients who have previously undergone breast irradiation or who are pregnant), in cases in which there is multifocal disease, or in cases in which there is strong patient preference. (Answer: E—Lumpectomy with axillary lymph node dissection and radiation therapy)

28. A 70-year-old woman presents for evaluation. Five years ago, she was diagnosed with stage II infiltrating ductal carcinoma of the right breast and was treated with lumpectomy and radiation. Over the past several weeks, she has experienced increasing fatigue and right upper quadrant pain. She denies having any bone pain. Physical examination reveals a hard, 3 cm palpable mass in the left breast (contralateral to her previous cancer); the examination is otherwise unremarkable. Results of laboratory testing include the following: AST, 78; ALT, 40; alkaline phosphatase, 200; total bilirubin, 1.3; and albumin, 3.0. CT of the abdomen shows three liver lesions that are consistent with metastases. Mammography reveals a spiculated mass in the left breast; a biopsy is performed, and the mass is found to be invasive ductal carcinoma that expresses both estrogen and progesterone receptors.

In this patient, first-line treatment should begin with which of the following?
A. Surgical resection of the breast nodule
B. Radiation of the left breast
C. Hormonal therapy
D. High-dose chemotherapy with autologous stem cell transplantation
E. Cytotoxic chemotherapy

Key Concept/Objective: To understand the management of metastatic breast cancer

This patient presents with metastatic breast cancer. In such patients, the goal of treatment is not cure but palliation of symptoms and improved survival. The role of surgery in advanced disease is limited to situations such as the resection of a solitary chest wall nodule or orthopedic stabilization. Radiation therapy is used to palliate bony lesions or brain metastases. Endocrine therapy is the first-line treatment of hormone-responsive metastatic disease. In postmenopausal women, therapy is initiated with an antiestrogen and followed by aromatase inhibitors, progestins, and androgens or estrogens, in that order, if these therapies prove ineffective. If the patient’s disease is unresponsive to hormonal therapy or is life-threatening, cytotoxic chemotherapy is initiated. Recent data have shown no advantage of CAF (cyclophosphamide, Adriamycin [doxorubicin], and fluorouracil) chemotherapy followed by high-dose alkylator therapy and stem cell support (autologous transplantation) as compared with CAF chemotherapy followed by CMF (cyclophosphamide, methotrexate, and fluorouracil) maintenance chemotherapy. (Answer: C—Hormonal therapy)

For more information, see Davidson NE: 12 Oncology: VII Breast Cancer. ACP Medicine Online (www.acpmedicine.com). Dale DC, Federman DD, Eds. WebMD Inc., New York, October 2004

Lung Cancer

29. A 44-year-old man comes to your clinic with questions about smoking cessation; he has smoked one and a half packs of cigarettes a day for the past 25 years. His father recently died of lung cancer, and your patient has decided to quit smoking. He wants to know about his risk of developing lung cancer.

Which of the following statements concerning the risk of this patient’s developing lung cancer is true?
A. His risk would be no higher if he had smoked two packs a day than if he had smoked one pack a day
B. If he does quit smoking now, in 20 years his risk of lung cancer will be the same as a man of the same age who never smoked
C. With regard to his risk of lung cancer, it makes no difference whether he stops smoking now or in 10 years
D. Even though he may quit smoking now, his risk of lung cancer will continue to rise with age
E. When assessing risk of lung cancer, it does not matter at what age he started smoking

Key Concept/Objective: To understand that age is a risk factor for the development of lung cancer and that, in former smokers, the risk of lung cancer increases with age

The following smoking factors have been identified as increasing lung cancer risk: aggregate amount of smoking; early onset of smoking; deeper inhalation; use of unfiltered cigarettes; high tar and nicotine content; and increasing age. A person who quits smoking does see a mortality benefit compared with someone of the same age who continues to smoke; however, the risk never returns to that of a lifelong nonsmoker. In addition, an American Cancer Society study showed that quitting at an earlier age (30 to 49 years) reduces risk more than quitting at a later age (50 to 64 years). (Answer: D—Even though he may quit smoking now, his risk of lung cancer will continue to rise with age)

30. A 52-year-old man comes to the office to ask if you know of any tests that will detect lung cancer at an early stage. He has a 45 pack-year history of smoking and continues to smoke.

For this patient, which of the following statements regarding screening for lung cancer is true?
A. Chest x-ray alone often fails to identify cancers that are potentially curable
B. On the basis of best evidence, spiral computed tomography is currently the recommended screening tool
C. Spiral CT combined with positron emission tomography (PET) has failed to detect early lung cancers
D. There are no data to suggest that screening for lung cancer improves survival

Key Concept/Objective: To understand that chest x-rays often miss curable cancers that are too small or indistinct to be detected

Clinical trails completed in the 1960s have shown the lack of efficacy of plain radiography as a screening tool for lung cancer. Spiral CT has been a promising technique, and at least one study has indicated a 5-year survival benefit to screening for lung cancer with spiral CT; however, other trials have failed to show any improvement in survival. The addition of PET scanning to CT of the chest has allowed the detection of early lung cancers in a few trials, but no randomized trials have been completed that have investigated the effect of the use of this combined diagnostic strategy on lung cancer mortality. There are currently insufficient data to allow an evidence-based recommendation regarding lung cancer screening with spiral CT, with or without the addition of PET scanning. (Answer: A— Chest x-ray alone often fails to identify cancers that are potentially curable)

31. A 63-year-old woman presents with a complaint of cough; the cough began 3 months ago. Evaluation further reveals a weight loss of 20 lb, left shoulder pain, generalized weakness, clubbing of the fingers, and loss of breath sounds in the left apex.

For this patient, which of the following statements regarding the clinical manifestations of lung cancer is false?
A. Cough is the most common symptom of a primary lung cancer
B. Weight loss is not a specific symptom of lung cancer
C. The left shoulder pain, if a manifestation of lung cancer, usually points to a tumor in the superior sulcus
D. Clubbing is a paraneoplastic syndrome resulting from nail bed swelling and deformity
E. Weakness may be the result of metastases or a paraneoplastic syndrome

Key Concept/Objective: To know that clubbing is a common manifestation of lung cancer and that it arises from periosteal swelling of the distal phalanges

Cough is the most commonly reported symptom of a primary lung tumor. Weight loss, although cause of suspicion of lung cancer in this patient, may also be found in a variety of other illnesses, including other cancers, chronic infections, and collagen-vascular disorders. Shoulder and arm pain may be caused by a tumor of the superior sulcus involving the eighth cervical and first thoracic nerves. Weakness can arise from several mechanisms in lung cancer, including metastases, anemia, electrolyte disturbances, and Lambert-Eaton syndrome. (Answer: D—Clubbing is a paraneoplastic syndrome resulting from nail bed swelling and deformity)

32. A 62-year-old woman with a 65 pack-year history of smoking comes to your office complaining of bloodtinged sputum. She has also experienced weight loss of 20 lb and left-sided chest pain. Chest x-ray reveals a 4 cm opacity in the left lower lobe.

Which of the following statements regarding the evaluation and staging of a possible lung cancer in this patient is false?
A. CT of the chest is indicated; images should include the adrenal glands
B. Staging of the cancer is more important than histologic type or degree of differentiation in determining prognosis
C. Bone scanning is indicated to evaluate for bony metastases
D. PET scanning, though not formally recommended at this point, may be helpful in identifying metastases, particularly those measuring more than 1 cm
E. The procedure of choice for biopsy of most suspected peripheral lung cancers is thoracotomy

Key Concept/Objective: To know that for most patients with peripheral lung masses, the procedure of choice for biopsy is video-assisted thoracoscopy (VATS) or needle biopsy

In the evaluation of a suspected lung cancer, the choice of biopsy technique depends on the site. If the lesion is centrally located or in the mediastinum, a bronchoscopy or mediastinoscopy is the procedure of choice; if the lesion is peripheral, VATS or CT-guided needle biopsy is preferred. If a metastatic site is identified, the patient should be offered the least invasive technique for diagnosis. Staging should include CT of the chest with visualization of the adrenals; CT of the head; and bone scanning. PET scanning is a promising technology and is being used in many centers; most data involve studies with lesions larger than 1 cm. The stage of the cancer is more important for prognosis than type or grade. (Answer: E—The procedure of choice for biopsy of most suspected peripheral lung cancers is thoracotomy)

For more information, see Crawford J: 12 Oncology: VIII Lung Cancer. ACP Medicine Online (www.acpmedicine.com). Dale DC, Federman DD, Eds. WebMD Inc., New York, February 2004

Prostate Cancer

33. A 65-year-old Chinese man comes to your indigent care clinic for routine health maintenance. He immigrated to the United States 35 years ago and works as a grocer. He has no complaints. Physical examination reveals poor dentition but is otherwise normal. Prostate examination reveals a smooth, normalsized, symmetrical prostate. A lipid panel shows the LDL cholesterol level to be 95 mg/dl and the HDL cholesterol level to be 50 mg/dl. The prostate-specific antigen (PSA) level is 1.3 ng/ml.

Which of the following statements regarding this patient’s risk of prostate cancer is true?

A. Advanced age is the most important risk factor for prostate cancer; most clinically detected prostate cancers are detected in the fifth and sixth decades of life
B. Chinese men have a moderate risk of prostate cancer
C. A diet high in red meat increases the risk of prostate cancer
D. Men with low testosterone levels who develop prostate cancer are more likely to develop lower-grade prostate cancer

Key Concept/Objective: To know that age, race, family history, diet, and hormone levels are important risk factors in the development of prostate cancer

Advancing age is the most obvious risk factor for prostate cancer; perhaps no other cancer is as age dependent. Most clinically detected prostate cancers are detected in the seventh and eighth decades of life. African Americans have the highest incidence of prostate cancer. The lowest incidence rates are in Japan and China. The dramatic differences between the Asian and Western diets possibly contribute to the significant difference in risk. Data from large cohort studies and case-control studies support the contentions that red meat, animal fat, and total fat consumption increase the risk of prostate cancer. In the Health Professionals Follow-up Study, men with lower testosterone levels who subsequently developed prostate cancer were more likely to develop higher-grade prostate cancer. (Answer: C— A diet high in red meat increases the risk of prostate cancer)

34. A 58-year-old white man presents to your clinic with a chief complaint of frequent urination. He awakes three or four times nightly to urinate. He denies having dysuria or hematuria. DRE reveals a smoothly enlarged prostate. Other results of the physical examination are normal. Results of a urinalysis are normal. The patient’s PSA level is 4.0 ng/ml.

For this patient, which of the following statements regarding screening for prostate cancer is true?

A. Most cancers detected by DRE are confined to the prostate and are usually curable
B. PSA is a glycoprotein with serine protease activity; it is a member of the kallikrein family and is produced only by malignant prostatic epithelial cells
C. Biopsy of the prostate in men who have moderately elevated PSA levels (i.e., PSA of 4 to 10 ng/ml) usually reveals prostate cancer
D. Prostate cancer is more likely when the total PSA level is high and the percentage of free PSA is low

Key Concept/Objective: To understand that the goal of screening for prostate cancer is to detect organ-confined prostate cancer that is potentially curable

Optimal screening for prostate cancer combines use of the PSA test and the DRE. Historically, DRE was used to screen for prostate cancer. DRE is inadequate, however, because its interpretation is highly variable, many cancers are not palpable, and most cancers detectable by DRE are not organ confined and therefore are incurable. PSA, a glycoprotein with serine protease activity in the kallikrein family, is abundant in semen, where it dissolves seminal coagulum. Both normal and malignant prostatic epithelial cells produce PSA; production may actually be higher in normal cells than in malignant cells. A problem with PSA-based screening is that an elevated PSA level lacks specificity. Despite the increased likelihood of prostate cancer in men with a moderately elevated serum PSA level (i.e., a level 4 to 10 ng/ml), biopsy usually reveals benign prostatic hyperplasia (BPH) rather than prostate cancer. Determination of the free PSA level (i.e., the percentage of PSA that is unbound to serum proteins) is also a potential means of distinguishing malignancy from benign hyperplasia. PSA derived from malignant epithelial cells tends to bind more avidly to serum proteins. Thus, in men with an elevated serum PSA level, cancer is more likely when the percentage of free PSA is low. (Answer: D—Prostate cancer is more likely when the total PSA level is high and the percentage of free PSA is low)

35. A 58-year-old white man with a PSA of 4.5 ng/ml and a normal DRE elects to undergo prostate biopsy to evaluate for prostate cancer. The biopsy is performed transrectally with ultrasound guidance, and multiple samples are obtained. The majority of tissue samples reveal BPH. However, one sample reveals prostate intraepithelial neoplasia (PIN).

For this patient, which of the following statements regarding the diagnosis of prostate cancer is true?
A. The most common prostate cancer is squamous epithelial cancer
B. The most commonly used grading system for prostate cancer is the Gleason grading system
C. PIN is a premalignant state; once diagnosed, further prostate biopsies are not indicated
D. Clinical staging of prostate cancer relies on CT imaging and bone scanning to determine degree of metastatic disease

Key Concept/Objective: To understand the role of multifactorial assessment in the diagnosis and stratification of patients with prostate cancer

The vast majority of prostate cancers are adenocarcinomas; small cell carcinomas, squamous cell carcinomas, and sarcomas are uncommon. The most commonly used grading system is the Gleason grading system, in which tumors are classified by the degree of disorganization of glandular structures. PIN represents a premalignant state; it is felt to predate true carcinoma and often coexists with carcinoma in the prostate gland. When biopsy reveals PIN but no actual cancer, further biopsies are warranted. The clinical stage of prostate cancer is based on the extent of disease assessed by palpation during DRE. Currently, prostate cancer is almost always diagnosed in men who have no radiographic evidence of metastases. The most clinically useful means of stratifying patients according to prognosis is through multifactorial staging—that is, through combined use of the clinical stage, the serum PSA level, and the Gleason score. (Answer: B—The most commonly used grading system for prostate cancer is the Gleason grading system)

36. A 76-year-old African-American man presents to the emergency department complaining of severe pain in the lower back and right hip. He reports that the pain has gotten gradually worse over the past month. He denies having other medical problems, and he has not seen a clinician for the past 10 years. He takes no medications. Results of physical examination are as follows: blood pressure, 130/60 mm Hg; heart rate, 88 beats/min; respiratory rate, 16 breaths/min; and temperature, 97.8° F (36.5° C). The lungs are clear. The cardiovascular examination is normal. The abdominal examination is benign with no organomegaly. Musculoskeletal examination reveals tenderness to palpation of the lumbar spine and right ischial tuberosity. Results of the neurologic examination are within normal limits. On DRE, the prostate is smooth and of normal size. Results of laboratory testing are as follows: WBC, 3,400 cells/mm3; hematocrit, 42%; platelet count, 450,000 cells/mm3. Bilirubin, aspartate aminotransferase (AST), and alanine aminotransferase (ALT) levels are normal, with an elevated alkaline phosphatase level of 240 mg/dl. The PSA level is 22 mg/dl.

For this patient, which of the following statements regarding the treatment of prostate cancer is true?
A. The only clear mortality benefit for radical prostatectomy is in patients with metastatic prostate cancer
B. External-beam radiation therapy may be preferable to radical prostatectomy for patients with localized prostate cancer because of the significantly decreased incidence of erectile dysfunction in patients treated with radiation
C. The standard treatment for patients with advanced prostate cancer is androgen ablation
D. Chemotherapy for hormone-resistant prostate cancer typically includes docetaxel plus prednisone; this treatment has been shown to improve quality of life but not decrease mortality

Key Concept/Objective: To understand the treatment of prostate cancer

In the United States, radical prostatectomy has been the standard treatment for prostate cancer and may offer the greatest chance of cancer control for patients with organ-confined prostate cancer. Radical prostatectomy is associated with urinary incontinence and erectile dysfunction; the frequency and severity of these side effects are a source of debate. In comparisons between radical prostatectomy and external-beam radiation therapy, men who undergo radical prostatectomy are more likely to have urinary incontinence or impotence, although significant decreases in sexual function are seen with both treatments; men who receive external-beam radiation therapy are more likely to suffer changes in bowel function. For patients with advanced prostate cancer, the standard initial treatment is androgen ablation, a therapeutic strategy that involves either lowering the production of testosterone or blocking its binding to the androgen receptor. Androgen ablation is achieved by a variety of strategies. Castration or diminishing testosterone production can be achieved surgically with orchiectomy or chemically with luteinizing hormone–releasing hormone agonists. Chemotherapy has a clear role in patients with hormone-refractory prostate cancer. Docetaxel plus prednisone is now the standard chemotherapy for men with metastatic prostate cancer. (Answer: C—The standard treatment for patients with advanced prostate cancer is androgen ablation)

37. A 65-year-old man attended a community-based health fair, where it was discovered that his PSA level was elevated; he was referred to your clinic. The patient states that he has always been healthy and that he has not seen a physician for years. On review of systems, he does note some frequency, hesitancy, and urgency of urination, which he has been experiencing for the past few years. DRE reveals a smooth, symmetrically enlarged prostate gland and brown guaiac-negative stool.

Which of the following is NOT associated with elevated PSA levels?
A. Prostate cancer
B. Benign prostatic hypertophy
C. Urethritis
D. Prostatitis

Key Concept/Objective: To know those conditions associated with elevated PSA levels

Screening for prostate cancer by use of PSA has led to an increase in the number of cases of prostate cancer diagnosed in recent years and has enhanced the ability to detect organconfined prostate cancer. However, some prostate cancers do not produce sufficient amounts of PSA to result in elevations in PSA serum levels, and there are several other conditions that can lead to elevations in PSA levels. These conditions include BPH, prostatitis, seminal ejaculation, and genitourinary instrumentation. Uncomplicated urethritis cannot lead to an elevation in PSA level. (Answer: C—Urethritis)

38. A 58-year-old patient of yours is diagnosed with prostate cancer after PSA levels were found to be elevated. Transrectal ultrasonography-guided biopsy is performed, and it is determined that the patient has adenocarcinoma of the prostate, with a Gleason score of 8. He asks you about his prognosis and the likelihood that he will die of his prostate cancer.

Which of the following patients is most likely to die of prostate cancer?
A. A 40-year-old man with a Gleason score of 8
B. A 40-year-old man with a Gleason score of 4
C. A 75-year-old man with a Gleason score of 8
D. A 75-year-old man with a Gleason score of 4

Key Concept/Objective: To know the key prognostic factors for patients with prostate cancer

The Gleason grading system is the most commonly used method of classifying prostate cancers. Tumors are graded from 1 (least malignant) to 5 (most malignant) on the basis of histologic findings. The two most common patterns observed are then added together to give a composite score. The majority of tumors are classified as Gleason 6 or 7, with grades of 7 or more considered high grade. In the Connecticut Tumor Registry, the two most important determinants of mortality from prostate cancer were age and Gleason grade. The patients at highest risk of dying of prostate cancer are those younger than 74 years and those with Gleason scores of 7 or higher. (Answer: A—A 40-year-old man with a Gleason score of 8)

39. A 70-year-old patient has been seeing you for treatment of hypertension for several years. Recently, you referred him to a urologist after a prostatic nodule was discovered on DRE and his PSA level was found to be elevated. The urologist diagnosed the patient as having prostate cancer on the basis of the results of a biopsy. He has offered the patient the option of radical prostatectomy or external-beam radiation therapy. The patient asks you about the side effects of these treatments.

Which of the following statements is false?
A. Most patients undergoing radical prostatectomy will become impotent
B. Erectile dysfunction occurs in a minority of patients undergoing external-beam radiation therapy
C. Radical prostatectomy is more likely to produce urinary incontinence or impotence than is external-beam radiation therapy
D. External-beam radiation therapy is more likely to result in a decline in bowel function than is radical prostatectomy

Key Concept/Objective: To know the major adverse side effects of radical prostatectomy and external-beam radiation therapy

Adverse side effects are frequent and significant in patients undergoing radical prostatectomy and external-beam radiation therapy. It is important to consider these adverse effects when deciding on a course of treatment. Stress urinary incontinence and erectile dysfunction are frequent in those undergoing radical prostatectomy. In fact, most patients undergoing the procedure are rendered impotent. The rates of both of these adverse effects are higher in patients undergoing radical prostatectomy than in those who receive externalbeam radiation therapy. However, erectile dysfunction still occurs in the majority of patients who undergo radiation therapy. In addition, a decline in bowel function is more common in patients receiving external-beam radiation therapy than in those who undergo radical prostatectomy. (Answer: B—Erectile dysfunction occurs in a minority of patients undergoing external-beam radiation therapy)

40. A 60-year-old man comes to you because he has heard there is a blood test for prostate cancer that he would like to be given. You explain that the decision to undergo screening for prostate cancer is not as simple as it might seem, and you want him to understand the screening process.

Which of the following statements should be included in your explanation to this patient?
A. Although one-time or repeated screening and aggressive treatment of prostate cancer may save lives and avert future cancer-related illness, we do not yet know this for certain
B. The available tests for prostate cancer (PSA and DRE) will sometimes indicate cancer when there is none (false positives) and will sometimes fail to detect cancer when it is present (false negatives)
C. A positive result on PSA or DRE will suggest that he should undergo invasive testing, such as transrectal ultrasound and prostate biopsy
D. Should he be found to have prostate cancer, he will want to consider aggressive therapy, and there is a small but finite risk of early death and a significant risk of chronic illness, particularly with regard to sexual and urinary function
E. All of the above

Key Concept/Objective: To understand the uncertainty surrounding screening for prostate cancer, and be able to communicate that uncertainty intelligibly to patients

There is disagreement as to whether men should be screened for prostate cancer. It is important to understand that it is not yet known whether screening for prostate cancer will help men live longer and that significant morbidity and mortality have been associated with the diagnostic and therapeutic procedures involved in screening. These facts should be conveyed to the patient to help him make an informed decision. (Answer: E—All of the above)

41. A 65-year-old man who is otherwise in excellent health comes to you for a second opinion regarding therapy for his recently diagnosed prostate cancer. His records show that his cancer was diagnosed on the basis of a screening PSA level of 5.0; his DRE result at the time was negative. Transrectal ultrasound revealed no apparent tumor, but four of six random biopsy specimens tested positive for cancer. His Gleason score is 7.

What is this patient’s risk of biochemical relapse (rising PSA) after local therapy?
A. Low, because his clinical tumor stage is T1c
B. Low, because his clinical tumor stage is T1c and his PSA level is less than 10
C. Intermediate, because his clinical tumor stage is T2b
D. Intermediate, because his Gleason score is 7
E. High, because a Gleason score of 7 indicates a high-grade tumor

Key Concept/Objective: To understand the clinical staging of prostate cancer

Clinical staging is based on the means of diagnosis and the size and location of the tumor. This patient’s tumor is stage T1c because it is not palpable and was found by biopsy after a positive PSA screening result. In such cases, risk of biochemical relapse (i.e., rising PSA level) is further assessed on the basis of clinical stage, PSA level, and Gleason score. Because this patient’s Gleason score is 7, his risk category is intermediate, even though his PSA level and clinical stage are relatively low. This case highlights the point that the tumors of patients whose Gleason scores are greater than 6 should be considered high grade. (Answer: D—Intermediate, because his Gleason score is 7)

42. From your assessment of risk for the patient in Question 41, what is the best advice that you can give him about treatment?
A. It is highly likely that his tumor is confined to the prostate, so radical prostatectomy, external-beam radiation, brachytherapy, and watchful waiting are all reasonable options
B. There is about a 50% chance of recurrence in 5 years, so radical prostatectomy is of no benefit
C. There is about a 50% chance of recurrence in 5 years, and radical prostatectomy is curative in 50% of patients with his profile
D. There is conclusive evidence that external-beam radiation is superior to radical prostatectomy in patients with his profile
E. It is very likely that his cancer has spread beyond the prostate, so the risks of radical prostatectomy are not justified; external-beam radiation with androgen ablation is a better choice

Key Concept/Objective: To understand that treatment options for patients with clinically localized prostate cancer depend on the patient’s risk of biochemical relapse

Because this patient is at intermediate risk, he has a 50% chance of relapse. Although this means that 50% of men with cancer of this stage will have clinically silent metastases, radical prostatectomy is curative in 50% of men in this risk group who undergo that procedure. There are as yet no data to suggest that prostatectomy or radiation therapy is of benefit with regard to mortality, and patients should be educated about the risks and benefits of both. (Answer: C—There is about a 50% chance of recurrence in 5 years, and radical prostatectomy is curative in 50% of patients with his profile)

43. The patient in Question 41 elects to undergo external-beam radiation. For 3 years, his PSA result is negative, then it rises to 2.8.

Which of the following treatment regimens has the best data to support it?
A. Salvage radical prostatectomy
B. Salvage radical prostatectomy and either surgical castration or chemical castration with LHRH agonists
C. Repeated external-beam radiation and either surgical castration or chemical castration with LHRH agonists
D. Antiandrogens, such as flutamide, bicalutamide, and nilutamide, and either surgical castration or chemical castration with LHRH agonists
E. Antiandrogens, such as flutamide, bicalutamide, and nilutamide

Key Concept/Objective: To know the therapies available for advanced prostate cancer and which to select, given the patient’s previous treatment

Because this patient initially underwent radiation therapy, he is not a candidate for salvage radical prostatectomy, because in such patients, the higher morbidity and mortality of the procedure outweigh the benefits. Neither would further radiation treatment be of benefit. From the available data, the best therapy would be to combine lowering of testosterone levels (which can be effected either surgically or through hormonal manipulation wih LHRH analogues) and treatment with antiandrogens, such as flutamide, bicalutamide, or nilutamide. (Answer: D—Antiandrogens, such as flutamide, bicalutamide, and nilutamide, and either surgical castration or chemical castration with LHRH agonists)

For more information, see Kantoff PW: 12 Oncology: IX Prostate Cancer. ACP Medicine Online (www.acpmedicine.com). Dale DC, Federman DD, Eds. WebMD Inc., New York, December 2004

Gynecologic Cancer

44. A 67-year-old nulliparous white woman presents to the clinic for evaluation of increasing abdominal girth and bloating; these symptoms have been occurring for several months and are associated with some abdominal discomfort. She previously underwent upper GI evaluation, the results of which were negative. She has not had a gynecologic examination for several years, but she denies having any vaginal bleeding or discharge. She also denies having any other relevant medical history, but her sister and her mother have breast cancer.

Which of the following gynecologic cancers is most likely for this patient?
A. Cervical cancer
B. Uterine cancer
C. Ovarian germ cell cancer
D. Ovarian epithelial cell cancer
E. Ovarian stromal cell cancer

Key Concept/Objective: To know the common gynecologic cancer risk factors and clinical presentation

Epithelial ovarian cancer occurs at a mean age of 60 years in the United States and is the most lethal of gynecologic tract tumors. In approximately 70% of women, the tumor has spread beyond the pelvis by the time of diagnosis. Temporary suppression of menstrual function is associated with a decreased risk of epithelial ovarian cancer. Nulliparity is associated with an increased risk of ovarian cancer. Women with advanced disease often note a progressive increase in abdominal girth and bloating for several months before they are diagnosed. These symptoms are caused by malignant ascites. Because of the nonspecific nature of the abdominal complaints, which are related to the presence of ascites and omental disease, many patients initially undergo an upper GI tract evaluation for a possible ulcer before the true nature of the illness is recognized. Two familial syndromes are associated with an increased risk of ovarian cancer. A family history suggestive of a genetic predisposition may be found in as many as 10% of patients with ovarian cancer. The breast-ovarian cancer syndrome, as the name implies, occurs in families whose members may have breast, ovarian, or both types of cancers. Mutations in the BRCA1 or BRCA2 genes are responsible for this syndrome. Ovarian germ cell tumors are derived from the oocyte and often occur in women younger than 20 years. Ovarian stromal cell tumors are derived from supporting elements such as granulosa or theca cells, which are normally responsible for sex steroid production. These tumors are characterized by their ability to secrete estradiol. Postmenopausal women present with breast tenderness, vaginal bleeding, or both. Women with cervical and uterine cancers most commonly present with vaginal bleeding or a grossly visible cervical lesion. (Answer: D—Ovarian epithelial cell cancer)

45. A 72-year-old woman presents with vaginal bleeding of 2 weeks’ duration. Her medical history is significant for right breast cancer, which was surgically cured and for which she takes tamoxifen. She has not had any other surgeries. Evaluation of this patient shows endometrial cancer.

Under which of the following circumstances is chemotherapy indicated for this patient?
A. After initial surgical intervention, the patient experiences rapidly progressive, symptomatic recurrence of the cancer
B. The cancer involves either ovary on initial evaluation
C. The cancer involves the uterus and cervix
D. The cancer involves distant sites such as lung or bone
E. All of the above

Key Concept/Objective: To understand the evaluation and treatment of uterine cancer

For rapidly progressive, symptomatic recurrence of uterine cancer, platinum-based chemotherapy is a reasonable treatment. There is no proven survival advantage associated with the use of adjuvant hormonal therapy (e.g., progestational agents) or chemotherapy, although these modalities may be useful for the management of systemic relapse. If the endometrial biopsy establishes the presence of uterine cancer, surgery for definitive resection and staging is the next most common step in management. For patients who are not optimal surgical candidates, primary radiotherapy is an option that can produce longterm survival in selected patients. Postoperative pelvic radiotherapy is considered when certain features confer an increased risk of local pelvic failure. These features include (1) deeply invasive, high-grade, early stage lesions (e.g., stage IC, grade III), especially with lymphovascular involvement, as these features are associated with an approximately 20% risk of occult pelvic lymph node involvement, and (2) cervical involvement (i.e., stage II), which confers an increased risk of vaginal vault recurrence. The use of tamoxifen, a drug traditionally thought of as an estrogen antagonist, is also associated with an increased uterine cancer risk. This is in part caused by the tissue-specific action of tamoxifen, which has antagonistic effects on proliferation of breast epithelium but agonistic effects on bone mineral density, lipid metabolism, and endometrial proliferation. However, the benefits of tamoxifen in the adjuvant breast cancer setting far outweigh the small risk of uterine cancer development. (Answer: A—After initial surgical intervention, the patient experiences rapidly progressive, symptomatic recurrence of the cancer)

46. A 50-year-old white woman with a history of fibrocystic breast disease and arthritis presents with abdominal pain of new onset. The patient describes the pain as an ache. The pain is mild and suprapubic and does not radiate. A urine dipstick evaluation performed in the clinic reveals that she has a urinary tract infection. The patient, however, is concerned that she may have ovarian cancer, because her mother died of ovarian cancer at age 59.

For this patient, which of the following statements is consistent with a diagnosis of ovarian cancer?
A. Approximately 30% of women present with advanced disease
B. Patients with advanced disease commonly complain of a progressive increase in abdominal girth and bloating
C. The primary lymphatic drainage site of ovarian cancer is the inguinal lymph nodes
D. The most common paraneoplastic syndrome associated with ovarian cancer is hypercalcemia

Key Concept/Objective: To understand the common clinical features of ovarian cancer

The ovary contains three distinct cell types, known as germ cells, stromal cells, and epithelial cells. The type of ovarian tumor that most commonly affects adult women, however, is derived from the epithelial cells that cover the ovarian surface. Epithelial ovarian cancer occurs at a mean age of 60 years in the United States and is the most lethal of gynecologic tract tumors. In approximately 70% of women, the tumor has spread beyond the pelvis by the time of diagnosis and cannot be completely resected at the time of exploratory laparotomy. Early-onset ovarian cancer that is restricted to the pelvis usually produces no signs or symptoms. Unfortunately, approximately 70% of women present with advanced disease that has extended beyond the pelvis to involve other areas, such as the upper abdomen (stage III) and the pleural space (stage IV). Women with advanced disease often note a progressive increase in abdominal girth and bloating for several months before they are diagnosed. These symptoms are caused by malignant ascites. The tumor can spread via the lymphatics to involve the para-aortic lymph node chain, which is the primary drainage site for the ovaries (as with the testes). In rare instances, patients may present with inguinal adenopathy as the first sign of disease. A large omental tumor cake can cause early satiety and weight loss as a result of gastric compression; however, weight loss is more commonly offset by the development of ascites. (Answer: B—Patients with advanced disease commonly complain of a progressive increase in abdominal girth and bloating)

47. A 65-year-old African-American woman presents with progressive increase in abdominal girth, bloating, and early satiety. A CT scan reveals a large left ovarian mass, ascites, and omental studding. The patient is sent for exploratory laparotomy and undergoes debulking. The tissue diagnosis is epithelial ovarian cancer.

For this patient, which of the following statements regarding the treatment of ovarian cancer is true?
A. Ovarian cancer relapse can be treated with the same chemotherapeutic drugs and has the same disease-free survival rate as those used for primary therapy
B. All women with ovarian cancer, even low-risk stage I disease, derive benefit from postoperative adjuvant chemotherapy
C. The mainstay of treatment for advanced-stage ovarian cancer is total abdominal hysterectomy with bilateral oophorectomy plus debulking and partial omentectomy, followed by a combination chemotherapy regimen containing a taxane and a platinum analogue
D. Platinum compounds, such as carboplatin and cisplatin, exert their cytotoxic effects by binding to and stabilizing the tubulin polymer during mitotic spindle formation

Key Concept/Objective: To understand the basic treatment for ovarian cancer

Exploratory laparotomy for evaluation of suspected ovarian cancer is typically performed with a vertical midline incision to provide adequate visualization of the upper abdomen. If the suspicion of epithelial ovarian cancer is confirmed by frozen section, a bilateral salpingo-oophorectomy and total abdominal hysterectomy are usually performed, along with a partial omentectomy. Other sites of tumor involvement are carefully evaluated with palpation and biopsy of the undersides of the diaphragm, the serosal surfaces of the bowel, and the paracolic gutters. The para-aortic lymph nodes are typically assessed when information about lymph node status would change patient management or when precise surgical staging is required to determine eligibility for protocol therapy. Finally, an attempt is made to remove as much tumor as possible at the time of initial surgery (debulking), because patients with residual tumor measuring less than 1 cm in diameter are more likely to respond to chemotherapy and have an improved survival rate. Some women with ovarian cancer have low-risk features that confer a 5-year survival rate of about 95%. Members of this low-risk group have stage IA or stage IB disease that is well-differentiated or moderately well-differentiated (i.e., grade I or II). These patients do not generally derive benefit from the use of postoperative adjuvant therapy. The goal of treatment of ovarian cancer relapse is palliation of symptoms. For patients with relapse detected only by the finding of an elevation of the CA-125 level, there is no convincing evidence that secondline, cytotoxic chemotherapy improves survival rate; however, it can compromise quality of life at a time when patients are feeling well. Platinum compounds, such as carboplatin and cisplatin, exert their cytotoxic effects by inducing DNA damage, primarily through the formation of intrastrand cross-links. (Answer: C—The mainstay of treatment for advancedstage ovarian cancer is total abdominal hysterectomy with bilateral oophorectomy plus debulking and partial omentectomy, followed by a combination chemotherapy regimen containing a taxane and a platinum analogue)

48. A 45-year-old white woman presents to your clinic with postcoital bleeding. She has not had a Papanicolaou (Pap) smear or pelvic examination in 15 years, despite the fact that she has multiple risk factors for cervical cancer. A Pap smear and pelvic examination are performed, and you diagnose her as having cervical cancer.

For this patient, which of the following statements regarding cervical cancer is false?
A. Cervical cancer is associated with human papillomavirus (HPV)
B. Risk factors for cervical cancer include multiple male sexual partners, sexual intercourse at an early age, and immunosuppression
C. Cervical cancer staging is based entirely on clinical criteria
D. Cervical cancer is a rapidly progressing malignancy

Key Concept/Objective: To understand the basic risk factors for and clinical features of cervical cancer

Invasive cervical cancer is uncommon in developed countries, partly because of the effectiveness of Pap smear screening. Nevertheless, it is estimated that in the United States in 2001, cervical cancer affected 12,900 women and caused approximately 4,400 deaths. Part of the success of Pap smear screening is due to the fact that this approach typically detects premalignant lesions, as opposed to invasive cancer. This unique feature makes it possible to eradicate precursor lesions before the development of frankly invasive cancers. In addition, the interval of time between the development of a precursor lesion and the occurrence of invasive disease may be several years, thus allowing many opportunities for the detection and eradication of premalignant disease. Cervical cancer is typically a disease of women in their fifth and sixth decades, whereas premalignant cervical lesions are often discovered in women younger than 40 years. This rather large gap in the age distribution between precursor lesions and invasive cancer is indicative of a long latency period for malignant transformation. Infection with HPV, most commonly subtypes 16, 18, 31, 33, and 35, is largely responsible for the development of precursor lesions and subsequent transformation to invasive disease. Not surprisingly, factors that predispose to transmission of this virus are associated with an increased risk of the development of cervical cancer. These high-risk factors are sexual intercourse at an early age, multiple male sexual partners, and male sexual partners who themselves have multiple partners. A history of smoking also confers a higher risk. Immunosuppression associated with either an underlying lymphoproliferative disorder such as Hodgkin disease or immunosuppressive drugs used in the prevention of allograft rejection also confers a higher risk of cervical cancer. In women with HIV infection, the immunosuppressive state associated with the infection increases the risk of development of cervical precursor lesions, although it is not clear whether the development of such lesions results in a higher incidence of invasive cervical disease. (Answer: D—Cervical cancer is a rapidly progressing malignancy)

For more information, see Cannistra SA: 12 Oncology: X Gynecologic Cancer. ACP Medicine Online (www.acpmedicine.com). Dale DC, Federman DD, Eds. WebMD Inc., New York, March 2004

Oncologic Emergencies

49. A 57-year-old woman with recently diagnosed stage IIIb squamous cell lung cancer presents to clinic with complaints of anorexia, fatigue, and diffuse abdominal discomfort. Additionally, her husband expresses concern that she has been more forgetful of late. Physical examination is largely unrevealing. Routine laboratory data are significant for a serum calcium level of 13.2 mg/dl. The patient is diagnosed with hypercalcemia of malignancy and is admitted to the hospital for management of this condition.

Which of the following treatments is contraindicated in patients with hypercalcemia of malignancy?
A. Thiazide diuretics
B. Loop diuretics
C. Infusion of I.V. normal saline
D. Bisphosphonates
E. Glucocorticoids

Key Concept/Objective: To understand the management of hypercalcemia of malignancy

Hypercalcemia of malignancy occurs in 10% to 20% of patients with cancer at some time during their illness. The treatment of patients with hypercalcemia of malignancy includes volume and electrolyte repletion, inhibition of bone resorption, and treatment of the underlying malignancy. Extracellular volume deficits exist in all patients with symptomatic hyercalcemia of malignancy. The single most important and urgent treatment is the infusion of normal saline to correct the extracellular volume deficit, increase the glomerular filtration rate, and, secondarily, increase renal calcium excretion. Loop diuretics should not be used until after the volume deficit has been fully corrected. Loop diuretics cause calciuresis and therefore may be effective in acutely decreasing calcium levels after volume repletion. Thiazide diuretics decrease renal calcium excretion and should be specifically avoided. The bisphosphonates offer an improved and simplified treatment of hypercalcemia of malignancy. The bisphosphonates have a high affinity for areas of high bone turnover, such as areas of bony involvement with malignancy, where they block osteoclast attachment to bone matrix and osteoclast recruitment and differentiation. (Answer: A— Thiazide diuretics)

50. A 72-year-old man with prostate cancer presents to the emergency department complaining of back pain. On further questioning, the patient reports having difficulty with ambulation for the past week, but he denies having bladder or bowel dysfunction. Physical examination reveals focal midthoracic vertebral body tenderness to percussion, 4/5 strength in the bilateral lower extremities, and normal patellar reflexes bilaterally. While plans for further evaluation are being made, the patient is treated with I.V. dexamethasone because of concern for epidural spinal cord compression.

Which of the following imaging modalities of the spine is recommended to evaluate for this complication?
A. Plain films
B. Radionuclide bone scan
C. Myelography
D. Gadolinium-enhanced MRI
E. CT with contrast

Key Concept/Objective: To know that the current recommendation for the radiographic evaluation of patients with possible epidural spinal cord compression is gadolinium-enhanced MRI

Epidural spinal cord compression should be suspected on the basis of the symptoms reported by the patient and the signs elicited by the physician on physical examination. Imaging of the spine provides the definitive diagnosis and the localization of the level of epidural spinal cord compression. Myelography, CT myelography, and gadolinium-enhanced MRI of the spine are the most sensitive and specific methods of evaluation. However, myelography is invasive and may be uncomfortable for the patient with severe bone pain; in addition, radiologists experienced in its interpretation may not be available. CT of the spine should not be performed, because its ability to scan the entire spinal axis efficiently and its sensitivity in identifying epidural disease are inferior to those of gadolinium-enhanced MRI. The current recommendation for the radiographic evaluation of patients with suspected epidural compression is gadolinium-enhanced MRI of the entire spinal axis. (Answer: D—Gadolinium-enhanced MRI)

51. A 63-year-old woman with no significant medical history presents to clinic with the report of facial swelling. Physical examination confirms the patient’s report of facial swelling and reveals distention of the jugular veins. A chest radiograph reveals superior mediastinal widening when compared with an earlier radiograph. Subsequently, contrast-enhanced CT of the chest reveals mediastinal lymphadenopathy and external compression of the superior vena cava (SVC) by an enlarged node.

What is the most appropriate step to take next in the management of this patient?
A. Immediate initiation of mediastinal irradiation
B. Fine-needle aspiration of an enlarged mediastinal lymph node
C. Positron emission tomography
D. Initiation of anticoagulation with warfarin
E. Thoracic MRI

Key Concept/Objective: To be aware that recent experience in adults suggests that SVC syndrome is not a true emergency and that histologic diagnosis should be quickly established and treatment promptly initiated

SVC syndrome is most commonly caused by extrinsic compression of the thin-walled, lowpressure SVC by a malignant mediastinal mass such as bronchogenic carcinoma (especially small cell lung cancer) and non-Hodgkin lymphoma. Before a histologic diagnosis is established, emergency treatment with mediastinal irradiation is only warranted in children and occasionally in adults who have mental status alteration, other life-threatening manifestations of increased intracranial pressure, cardiovascular collapse, or evidence of upper airway obstruction. In the absence of such conditions, as with this patient, the next step in the management of SVC syndrome should focus on efforts to obtain a histologic diagnosis of the underlying condition so that appropriate therapy may be initiated. Obtaining further radiologic studies will not assist in making a histologic diagnosis and are therefore not the most appropriate next step in the management of this patient. (Answer: B—Fine-needle aspiration of an enlarged mediastinal lymph node)

52. A 39-year-old woman with a history of T2N2M0 breast cancer presents with dyspnea that has been increasing over the past 3 days. She has no pleuritic chest pain. She received six cycles of chemotherapy with cyclophosphamide, doxorubicin, and fluorouracil; the chemotherapy ended 6 months ago. On examination, her blood pressure is 80/60 mm Hg and her pulse is 120 beats/min. Her jugular venous pressure is elevated to 10 cm, and her lungs are clear. She has trace edema in the lower extremities. Chest x-ray reveals cardiomegaly with no pleural effusions or infiltrates.

What is the most likely diagnosis for this patient?
A. Doxorubicin-induced cardiomyopathy
B. Pericardial tamponade
C. Pulmonary embolus
D. Tension pneumothorax
E. Superior vena cava syndrome

Key Concept/Objective: To be able to recognize the typical signs and symptoms of pericardial tamponade

This patient presents with dyspnea, hypotension, and elevated neck veins. She does not have signs or x-ray findings of left ventricular failure. She has significant risk of malignant pericardial disease (T2N2M0 breast cancer). The most likely diagnosis is cardiac tamponade caused by malignant pericardial disease. She is at risk for doxorubicin-induced cardiomyopathy, but her physical examination and chest x-ray do not show evidence of leftsided heart failure. A pulmonary embolus could cause dyspnea, hypotension, and acute right heart failure. Her cardiomegaly is not consistent with an acute pulmonary embolus. Both doxorubicin-induced cardiomyopathy and pulmonary embolus are important considerations, though these are less likely in this patient than pericardial tamponade. An echocardiogram would be the most useful test to sort out these diagnostic possibilities. Tension pneumothorax and superior vena cava syndrome are unlikely because of the chest x-ray results. (Answer: B—Pericardial tamponade)

53. A 63-year-old woman with acute myeloid leukemia (AML) after chemotherapy presents for follow-up. She has been tired but is not having any fevers or bleeding. On examination, the patient is pale, with a grade II/VI systolic ejection murmur at the left sternal border. Laboratory test results are as follows: Hb, 8; HCT, 24; platelet count, 15,000; WBC, 1.8.

What would you recommend for this patient?
A. Platelet transfusions to maintain a platelet count greater than 20,000
B. Platelet transfusions to maintain a platelet count greater than 50,000
C. Platelet transfusion if patient has bleeding or if the platelet count drops below 10,000
D. Platelet transfusion if patient experiences bleeding

Key Concept/Objective: To understand the risks associated with platelet transfusions and the basis for recommending platelet transfusions in leukemia patients with thrombocytopenia

This patient has thrombocytopenia related to her AML, her chemotherapy, or both. Although the best time to intervene with platelet therapy is somewhat controversial, there is a growing consensus that treatment should be initiated if the platelet count is low and the patient is bleeding and that prophylactic platelet therapy should be started only when the count is below 10,000. (Answer: C—Platelet transfusion if patient has bleeding or if the platelet count drops below 10,000)

54. A 26-year-old woman with Hodgkin disease presents after her third cycle of chemotherapy. She had her last chemotherapy session 10 days ago. She does not have cough, pleuritic chest pain, or abdominal pain. Her temperature is 103.8° F (39.8° C); her pulse is 100 beats/min; her respiration rate is 18 breaths/min; her skin is without rashes; her chest is clear; she is without heart murmur; and her abdomen is soft. Chest x-ray shows no infiltrates. The WBC is 0.7 (200 neutrophils, 400 lymphocytes, 100 basophils).

What would you recommend next for this patient?
A. Blood, sputum, and urine cultures
B. Blood, sputum, and urine cultures; begin amoxicillin-clavulanate
C. Blood, sputum, and urine cultures; begin ceftriaxone
D. Blood, sputum, and urine cultures; begin ceftazidime and tobramycin
E. Blood, sputum, and urine cultures; begin granulocyte colony-stimulating factor

Key Concept: To understand the approach to the treatment of patients with neutropenic fevers

This patient with chemotherapy-induced neutropenia has a fever, and her examination does not suggest a cause of this fever. She should receive antibiotics that provide coverage of gram-negative rods, including Pseudomonas aeruginosa, as well as gram-positive coverage. There is no one optimal treatment regimen. Studies have shown that monotherapy with imipenem and ceftazidime have been as effective as two-drug combinations. In this patient, amoxicillin-clavulanate would not provide broad enough coverage against gramnegative rods. Several studies have suggested that oral quinolone therapy may be an option in selected patients. Ceftriaxone would not offer Pseudomonas coverage. (Answer: D—Blood, sputum, and urine cultures; begin ceftazidime and tobramycin)

For more information, see Carlson RW: 12 Oncology: XII Oncologic Emergencies. ACP Medicine Online (www.acpmedicine.com). Dale DC, Federman DD, Eds. WebMD Inc., New York, December 2002

Sarcomas of Soft Tissue and Bone

55. A 40-year-old white woman presents to your office complaining of a painful, rapidly enlarging mass on her chest. The mass has been present for 1 month. She has a history of Hodgkin disease, for which she underwent radiotherapy 20 years ago. Biopsy reveals osteosarcoma.

Which of the following statements is true regarding this patient?
A. Sarcomas arising in radiation ports are more resistant to chemotherapy
B. Her risk of developing a sarcoma after radiotherapy was 10%
C. Her case is unusual in that most cases of sarcoma related to radiotherapy occur approximately 40 years after exposure to radiation
D. The most common type of sarcoma associated with previous radiotherapy is not osteosarcoma but rather leiomyosarcoma

Key Concept/Objective: To know that radiotherapy is a risk factor for sarcoma

Patients who have undergone radiotherapy are at increased risk for developing sarcoma. Sarcomas arising in radiation ports are more resistant to chemotherapy. The risk of a secondary sarcoma after radiation exposure is substantially less than 1%, and the patient is typically exposed to radiation 4 to 20 years before the development of sarcoma. Most radiation-associated sarcomas are osteosarcomas. (Answer: A—Sarcomas arising in radiation ports are more resistant to chemotherapy)

56. A 55-year-old man presents for evaluation of an enlarging mass in his left upper extremity. He first noticed the mass 3 months ago. It is painless, and the only reason he is concerned is because it continues to enlarge. Results of CT scanning and biopsy are consistent with soft tissue sarcoma.

Which of the following statements regarding this patient is true?
A. The grade of the tumor is based on the amount of necrosis seen on imaging
B. His prognosis would be better if he had an intra-abdominal or retroperitoneal tumor
C. The 5-year survival rates for patients with sarcomas (excluding intraabdominal and retroperitoneal sarcomas) are similar when corrected for grade, size, and depth
D. Patients with high-grade tumors have an unusually poor prognosis, even when the tumors are less than 5 cm

Key Concept/Objective: To understand the basic principles regarding the grading and staging of sarcomas, as well as prognosis

Staging of sarcomas is based on tumor size, grade, and depth. The 5-year survival rates for patients with soft tissue sarcomas arising in different anatomic sites are similar when corrected for grade, size, and depth, except for intra-abdominal and retroperitoneal tumors, which tend to be large and to invade vital organs, even if they are low grade. Patients with low-grade, superficial tumors tend to do well if the tumors are adequately resected. Even patients with high-grade tumors have a good prognosis if the tumors are less than 5 cm in diameter. Tumor grade is based largely on the number of mitoses per high-powered field (magnification, 10×). (Answer: C—The 5-year survival rates for patients with sarcomas [excluding intra-abdominal and retroperitoneal sarcomas] are similar when corrected for grade, size, and depth)

57. A patient is referred to you by his dermatologist for evaluation of a soft tissue mass on his leg. You obtain an MRI of the primary lesion and a CT scan of the chest, because you are concerned about the possibility of soft tissue sarcoma. You recommend that an incisional biopsy be performed for definitive diagnosis. The patient wants to know how you would treat such a tumor.

Which of the following is true regarding the general treatment of soft tissue sarcomas?
A. Soft tissue sarcomas are usually well encapsulated and are seen to have clear margins on resection
B. Local control of soft tissue sarcomas consists of surgical resection, often with radiotherapy
C. Chemotherapy is never indicated for soft tissue sarcomas
D. The presence of necrosis on MRI suggests a low-grade sarcoma

Key Concept/Objective: To understand the basic principles of sarcoma therapy

The goals of the treatment of sarcomas are local and systemic control of the sarcoma; preservation of the extremity or organ function; and quality of life. Local control of a soft tissue sarcoma is generally achieved by surgical resection, which is often combined with radiotherapy. Low-grade tumors push aside contiguous structures, whereas high-grade tumors invade adjacent organs and have large areas of necrosis. Soft tissue sarcomas grow along histologic planes and are usually pseudoencapsulated (i.e., microscopic projections of tumor extend beyond the apparent tumor capsule). Any excision that merely “shells out” the apparently encapsulated tumor generally leaves behind microscopic residual tumor, resulting in regrowth of the tumor 80% of the time. Multimodality therapy, including chemotherapy, is routine therapy for osteosarcomas, rhabdomyosarcomas, and Ewing sarcoma. (Answer: B—Local control of soft tissue sarcomas consists of surgical resection, often with radiotherapy)

58. A 48-year-old black woman presents to your office for follow-up. She was recently admitted to the hospital for upper gastrointestinal bleeding. An esophagoduodenoscopy was performed. A mass was noted in her stomach, and a biopsy was performed. The pathology report identified a gastrointestinal stromal tumor (GIST).

Which of the following statements is true regarding GISTs?
A. The prognosis is better when the tumor is in the small intestine than when it is in the stomach or esophagus
B. GISTs have muscle markers similar to those of leiomyosarcomas, including muscle or Schwann cell (S-100) markers
C. GISTs are only found in the stomach
D. GISTs express c-kit (a proto-oncogene that encodes transmembrane tyrosine kinase receptors) with a mutation that causes the receptor to be constitutively active; imatinib (Gleevec) has shown promise in the treatment of GISTs

Key Concept/Objective: To understand GISTs

GISTs occur predominantly in middle-aged patients. Approximately 70% occur in the stomach, 20% occur in the small intestine, and less than 10% occur in the colon, esophagus, and rectum. Survival correlates with tumor location. The best prognoses are associated with tumors of the esophagus and stomach; the worst prognoses are associated with tumors occurring in the small intestine. GISTs frequently lack muscle and Schwann cell markers that are typical of leiomyosarcomas found in other anatomic sites. The protooncogene c-kit encodes a transmembrane tyrosine kinase receptor located on the long arm of chromosome 4. Mutations of c-kit cause the receptor to be activated constitutively without its ligand. Imatinib is an oral, relatively specific inhibitor of three tyrosine kinase receptors. It is used in the treatment of GISTs. (Answer: D—GISTs express c-kit [a proto-oncogene that encodes transmembrane tyrosine kinase receptors] with a mutation that causes the receptor to be constitutively active; imatinib [Gleevec] has shown promise in the treatment of GISTs)

For more information, see Raftopoulos H, Antman KH: 12 Oncology: XIII Sarcomas of Soft Tissue and Bone. ACP Medicine Online (www.acpmedicine.com). Dale DC, Federman DD, Eds. WebMD Inc., New York, November 2003

Bladder, Renal, and Testicular Cancer

59. A 60-year-old man presents to the walk-in clinic with fever and malaise of 3 days’ duration. He was recently diagnosed with transitional cell bladder cancer after presenting with microscopic hematuria. Staging protocol found no evidence of metastatic disease. An endoscopic resection of the tumor showed superficial bladder cancer that was restricted to the bladder mucosa. After transurethral resection, the patient was started on weekly intravesical infusions of bacillus Calmette Guérin (BCG); he has received five such infusions so far. On physical examination, the patient has fever but is otherwise normal. He has had macroscopic hematuria for 2 days. A chest x-ray is normal.

Of the following, which is the most likely cause of this patient’s symptoms?
A. Recurrent bladder cancer
B. BCG infusion
C. Early pneumonia
D. Febrile neutropenia

Key Concept/Objective: To know the possible side effects of BCG infusion

Superficial bladder cancer, which constitutes about 80% of incident cases, is restricted to the level of the bladder mucosa and lamina propria. For superficial bladder cancer, the initial treatment is careful and thorough endoscopic resection of the tumor or tumors. In patients at high risk for recurrence, BCG may be infused into the bladder through a catheter as an adjuvant to transurethral resection. The mechanism of action is incompletely understood, but it appears to be based on local immunologic response. Side effects of intravesical BCG include dysuria, urinary frequency, hematuria, and a flulike syndrome. More significantly, because BCG is an attenuated mycobacterium, it can produce local, regional, and systemic infections. Granulomatous infections can occur at extravesical sites, including the prostate, epididymis, testes, kidney, liver, and lungs. BCG sepsis is the most serious complication and can be life-threatening. Systemic involvement is treated with triple-antibiotic antituberculous therapy for 6 months. (Answer: B—BCG infusion)

60. A 30-year-old woman comes to your clinic complaining of right-side flank pain and macroscopic hematuria. You make a presumptive diagnosis of nephrolithiasis and order a CT scan, which shows no stones; however, the report describes three masses in the right kidney, the largest measuring 5 × 4 cm; two masses are seen in the left kidney, the larger measuring 3 × 3 cm. A 3 × 3 cm pancreatic cyst is also found. You call your patient to discuss these CT findings, and you ask her about her family history. She says her sister had a brain tumor that caused her to have gait problems, and a brother had kidney cancer.

What is the most likely diagnosis for this patient?
A. Metastatic pancreatic cancer
B. Familial papillary renal cell cancer
C. Polycystic kidney disease with malignant transformation
D. Von Hippel-Lindau (VHL) disease

Key Concept/Objective: To know the manifestations of VHL disease

Most renal cell carcinomas occur sporadically, but about 4% of cases present in an inherited pattern. Such familial cancers include VHL disease and familial papillary renal cell cancers. Approximately 1.6% of renal cell cancers are part of the autosomal dominant VHL disease, which is also characterized by retinal and central nervous system hemangioblastoma, pheochromocytoma, and pancreatic cyst. Compared with sporadic cases, renal cell cancer in the VHL syndrome tends to be multifocal and bilateral and to appear at a younger age. This patient’s age, the presence of a strong family history, and the findings on the CT scan make the likelihood of VHL disease very high. Polycystic kidney disease is not a risk factor for renal cell cancer. However, a threefoldto sixfold-higher incidence of renal cell cancer has been found in the chronic dialysis population as well as in renal transplant recipients, presumably because of the development of acquired cystic kidney disease. (Answer: D—Von Hippel-Lindau [VHL] disease)

61. A 25-year-old man comes to the clinic after finding a painless mass on his right testicle. He has no symptoms except for mild pain on the right flank and headache of new onset. The physical examination shows a 2 × 2 cm solid, hard mass on the right testicle. Physical examination shows no lymphadenopathy or other abnormalities. An ultrasound is obtained, which shows a mass on the right testicle consistent with a tumor. A chest x-ray shows multiple “cannonball” lesions in both lungs. A pelvic CT scan shows diffuse retroperitoneal lymphadenopathy. A magnetic resonance imaging scan of the brain also shows multiple masses consistent with metastatic disease.

On the basis of these findings and the overall prognosis, how would you approach this patient?
A. Refer to surgical oncology and radiation oncology for orchiectomy and radiotherapy
B. Explain the bad prognosis of his extensive disease and refer to palliative care
C. Refer to oncology to start chemotherapy
D. Refer to surgical oncology and radiation oncology for orchiectomy with radical retroperitoneal lymph node dissection and radiotherapy

Key Concept/Objective: To outline the management of metastatic testicular cancer

For testicular cancer patients with lymph node metastases measuring more than 5 cm in diameter and for those with visceral metastases (e.g., liver, lung, bone), the treatment of choice is systemic chemotherapy. The early combination of cisplatin, vinblastine, and bleomycin (the PVB regimen) produced cures in up to 70% of cases. Subsequently, a less toxic regimen, in which vinblastine was replaced by etoposide, was tested and shown to be equivalent to the PVB regimen. Metastatic diseases in the brain and liver, and possibly in bone, are associated with a worse prognosis. However, it must be emphasized that cure is possible in patients in the worst prognostic groups; even patients with brain metastases may be cured by aggressive multimodality treatment. Orchiectomy, radiotherapy, and radical retroperitoneal lymph node dissection are used in early stages of testicular cancer, when the disease is confined to the testicles or when the lymph node metastases are smaller than 5 cm. (Answer: C—Refer to oncology to start chemotherapy)

62. A 32-year-old man with a recently diagnosed metastatic seminoma is started on a regimen of cisplatin, vinblastin, and bleomycin. His initial human chorionic gonadotropin (hCG) level is 80 mIU/ml. One day after starting his chemotherapy, the hCG level rises to 100 mIU/ml, and the following day it is 130 mIU/ml.

On the basis of these laboratory findings, what would be the next step in the treatment of this patient?
A. Reaccess the hCG level in 3 or 4 days; you expect it to be decreased by then
B. Consider switching the patient’s regimen to cisplatin, etoposide, vinblastine, and bleomycin
C. Ask the pathologist to reevaluate the biopsy, considering the possibility that the correct diagnosis is choriocarcinoma and not seminoma
D. Change the regimen by alternating cisplatin, vincristine, methotrexate, and bleomycin with actinomycin D, cyclophosphamide, and etoposide

Key Concept/Objective: To describe the usual response in testicular tumor markers to chemotherapy

About 30% of seminomas include syncytiotrophoblastic giant cells, and these may produce hCG. The circulating testicular tumor markers hCG and a-fetoprotein should be measured as part of the initial diagnostic workup and in the monitoring of therapy. The first specimen should be obtained before primary surgery. hCG has a half-life of 24 to 36 hours. Failure of a circulating marker to decline in accordance with normal half-life gradients after orchiectomy suggests the continuing release of the marker into the blood by occult metastatic disease. In patients with metastatic disease, during chemotherapy, the release of markers from dying cancer cells may result in a transient elevation of blood levels, after which they will decline according to half-life gradients. Thus, serial measurements should be taken to determine whether the patient is responding adequately to treatment. The other regimens listed among the choices have been used for patients with poor risk or who have previously undergone treatment for metastatic disease. An extremely high hCG level (i.e., one in the thousands) is typical in choriocarcinoma. (Answer: A— Reaccess the hCG level in 3 or 4 days; you expect it to be decreased by then)

For more information, see Raghavan D: 12 Oncology: XIV Bladder, Renal, and Testicular Cancer. ACP Medicine Online (www.acpmedicine.com). Dale DC, Federman DD, Eds. WebMD Inc., New York, May 2003

Chronic Lymphoid Leukemias and Plasma Cell Disorders

63. A 70-year-old woman presents for routine check-up. She has hypertension that is well controlled on medication. She has no specific complaints but is fatigued. On physical examination, a few anterior cervical and axillary lymph nodes are found to measure 2 cm in diameter. Laboratory results are as follows: WBC, 106,000/µl, with a lymphocyte predominance; hematocrit, 39%; platelet count, 160,000/µl. Other laboratory results are normal. A CT scan of the abdomen reveals some periaortic lymphadenopathy. Flow cytometry of her peripheral blood reveals that most of the cells express CD20, CD23, and CD5 antigens.

What is the most appropriate treatment for this patient at this time?
A. Chlorambucil
B. Intravenous ?-globulin (IVIG)
C. Chlorambucil and prednisone
D. Fludarabine
E. Observation

Key Concept/Objective: To know the appropriate therapy for CLL

CLL, a malignancy of the B cells, is the most common of all the leukemias. It generally affects older adults; it affects men more than women and is more common in Jewish people of Eastern European descent. Patients are usually asymptomatic at the time of diagnosis; the lymphocytosis is usually noted on routine screening. The etiology of CLL is unknown. CLL is a clonal expansion of mature lymphocytes. Diagnosis can be confirmed by flow cytometry because CLL cells usually express normal CD19 and CD20 antigens, but they also express activation antigens CD5 and CD23. Disease staging is based on the Rai or the Binet classification system, and prognosis is related to the stage of the disease. Of the currently available therapies for CLL, none are curative, and no survival advantage has been shown with treatment of early stage disease at diagnosis. Therapy should therefore be initiated only when indicated by symptoms: fever, chills, weight loss, severe fatigue, bone marrow failure with anemia or thrombocytopenia, massive lymphadenopathy or hepatosplenomegaly, or recurrent infections. Although most patients with CLL have hypogammaglobulinemia, IVIG fails to protect patients from infections; it has no influence on survival and is not cost-effective. In this patient, who is otherwise doing well, there is no role for chemotherapy. (Answer: E—Observation)

64. Which of the following is NOT associated with chronic lymphocytic leukemia (CLL)?
A. Richter syndrome
B. Hypogammaglobulinemia
C. Increased risk of bacterial and fungal infection
D. Chronic myelogenous leukemia (CML)
E. Transformation to prolymphocytic leukemia (PLL)

Key Concept/Objective: To know the complications of CLL

CLL is a clinically heterogeneous disorder; survival is variable. Some patients live for years after diagnosis, and some die within months. The clinical course depends on the stage of disease at diagnosis; prognostic risk factors include male sex, black race, poor performance status, and older age. In addition, short lymphocyte doubling time is also predictive of poor outcome. In some patients with CLL, the clinical course is complicated by progressive conditions, secondary malignancies, immune abnormalities, and infections. Progressive conditions include Richter syndrome, in which patients develop worsening lymphadenopathy, hepatosplenomegaly, fever, abdominal pain, weight loss, and anemia. Patients who develop Richter syndrome do not respond well to therapy, and survival is short. PLL is the second most common transformation in CLL; however, most cases of PLL arise de novo. Compared with patients with de novo disease, patients with PLL that arises through transformation tend to be younger, and they have less marked lymphocytosis. Other transformations in CLL include acute lymphoblastic leukemia, multiple myeloma, and Hodgkin lymphoma. CML is not a known complication of CLL. (Answer: D—Chronic myelogenous leukemia [CML])

65. A 64-year-old man presents with fatigue and weakness. His physical examination is normal except for splenomegaly. A complete blood count reveals pancytopenia, with lymphocyte predominance and normal red cells. A bone marrow aspirate was dry. The bone marrow biopsy is pending.

Which of the following is the most likely diagnosis for this patient

A. CLL
B. Hairy-cell leukemia (HCL)
C. CML
D. Multiple myeloma
E. Myelofibrosis

Key Concept/Objective: To understand the diagnosis and differential diagnosis of HCL

HCL is a rare B cell neoplasm that most often occurs in older men. Patients usually present with symptoms related to impairment of the bone marrow, such as infections and bleeding, although they may be asymptomatic. The physical examination is usually only significant for splenomegaly; laboratory studies typically reveal pancytopenia. The bone marrow aspirate is often dry secondary to fibrosis; biopsy may be needed to make the diagnosis. In this patient, CLL and CML are unlikely because of the leukopenia. Multiple myeloma is unlikely, given the dry marrow aspirate. Myelofibrosis is unlikely secondary to the normal red cell morphology. Treatment of HCL is indicated only if there is worsening splenomegaly or lymphadenopathy or if there are more than 20,000 hairy cells/µl. In the past, splenectomy was the treatment of choice; however, splenectomy is used less commonly today. Although splenectomy may improve symptoms, it does not affect the disease itself. The current treatment of choice is either cladribine or the purine analogue pentostatin; both of these agents have been shown to induce remission in 80% of patients, and the choice is based on physician preference. (Answer: B—Hairy-cell leukemia [HCL])

66. A 76-year-old man presents with back pain and malaise. Initial laboratory results are as follows: WBC, 3,000/µl; hematocrit, 28%; platelet count, 200,000/µl. Serum chemistries are otherwise normal. Serum protein electrophoresis (SPEP) reveals a monoclonal protein level of 3.8 g/dl; immunoelectrophoresis revealed these monoclonal proteins to be IgG-?. Bone marrow evaluation reveals sheets of dysplastic plasma cells, and skeletal survey reveals osteolytic lesions in the skull and vertebrae.

In addition to melphalan and prednisone, which of the following would be best to add to this patient’s regimen?
A. Granulocyte colony-stimulating factor (G-CSF)
B. Doxorubicin
C. Fludarabine
D. Pamidronate
E. Interferon alfa

Key Concept/Objective: To understand the role of bisphosphonates in the treatment of multiple myeloma

This patient has most of the classic symptoms of multiple myeloma, including plasma cell infiltration of the bone marrow, osteolytic bone lesions, anemia, and an M protein level of greater than 3.5 g/dl on SPEP. Chemotherapy with melphalan and prednisone is a reasonable therapeutic option because this combination has been shown to have a higher response rate than monotherapy with either drug. In patients who have bone disease, pamidronate is added to provide protection against skeletal complications; this approach appears to improve quality of life and possibly provides a survival advantage. Therefore, current recommendations are to add bisphosphonates such as pamidronate to the regimens for all patients with evidence of bone involvement. (Answer: D—Pamidronate)

For more information, see Cheson BD: 12 Oncology: XV Chronic Lymphoid Leukemias and Plasma Cell Disorders. ACP Medicine Online (www.acpmedicine.com). Dale DC, Federman DD, Eds. WebMD Inc., New York, February 2002

Acute Leukemia

67. A 74-year-old white woman presents to you in clinic for routine follow-up. She has been your patient for several years. Her medical problems consist of hypertension and mild degenerative joint disease, for which she is receiving hydrochlorothiazide and nonsteroidal anti-inflammatory drugs (NSAIDs) as needed. On her last visit, her only complaint was of increasing fatigue, which she had been experiencing for several months. Results of routine laboratory tests at that time were as follows: white blood cell count, 7,500 cells/µl; hematocrit, 26%; mean cell volume, 96 fl; and platelet count, 485,000/µl. Follow-up laboratory studies revealed no vitamin B12 or folate deficiencies. Results of iron studies were also normal. Repeat laboratory values today reveal a persistent macrocytic anemia and an elevated platelet count. You schedule a bone marrow biopsy to further evaluate the patient’s anemia; it reveals the presence of monolobulated and bilobulated micromegakaryocytes.

The presence of monolobulated and bilobulated micromegakaryocytes characterizes which of the following chromosomal abnormalities?
A. A deletion of the long arm of chromosome 5
B. A translocation of chromosomes 15 and 17
C. A translocation of chromosomes 9 and 22
D. A deletion of the long arm of chromosome 9

Key Concept/Objective: To know the clinical presentation of myelodysplastic syndrome (MDS)

Considerable data suggest that MDS results from combined defects of both stroma and hematopoietic stem cells. Several clinical syndromes that may have a more predictable natural history can now be defined. For example, a deletion of the long arm of chromosome 5 can be detected in some older patients, especially women, with a macrocytic, refractory anemia (RA). The platelet count is typically normal or elevated. The bone marrow picture in the RA with 5q– syndrome is characterized by the presence of monolobulated and bilobulated micromegakaryocytes. Two thirds of these patients have RA or RA with ringed sideroblasts (RARS), and the remainder have RAEB (RA with excess of blasts). In those patients who have a del(5q) as their sole cytogenetic abnormality, MDS tends to follow a more benign course, although progression to acute myeloid leukemia (AML) may occur. (Answer: A—A deletion of the long arm of chromosome 5)

68. A 62-year-old woman well known to you comes to see you in clinic. Since the last time you saw her, she was admitted to the hospital and diagnosed with acute leukemia. She has been followed by a local hematologist and has undergone remission-induction chemotherapy. She is scheduled to begin postinduction consolidation therapy. She explains that she and the specialist are working toward a “complete remission” (CR) and wants to know if that means she will be cured.

Which of the following definitions of CR is most accurate?
A. Full recovery of normal peripheral blood counts; blast cells are undetectable in the bone marrow
B. Full recovery of normal peripheral blood counts; bone marrow cellularity with less than 5% residual blast cells
C. Full recovery of normal peripheral blood counts; bone marrow cellularity with less than 10% residual blast cells
D. Full recovery of normal peripheral blood counts; bone marrow cellularity with less than 10% residual blast cells for a minimum of 1 year

Key Concept/Objective: To understand the concept of CR in leukemia patients

The goal of remission-induction chemotherapy is the rapid restoration of normal bone marrow function. The term complete remission is reserved for patients who have full recovery of normal peripheral blood counts and bone marrow cellularity with less than
5% residual blast cells. Induction therapy aims to reduce the total-body leukemia cell population from approximately 1012 cells to below the cytologically detectable level of about 109 cells. The leukemia cells in some patients have high levels of primary drug resistance and will be refractory to courses of remission-induction chemotherapy. It is assumed, however, that even in CR a substantial burden of leukemia cells persists undetected, leading to relapse within a few weeks or months if no further therapy is administered. Postinduction or remission consolidation therapy, usually comprising several additional courses of chemotherapy, is designed to eradicate residual leukemia, allowing the possibility of cure. (Answer: B—Full recovery of normal peripheral blood counts; bone marrow cellularity with less than 5% residual blast cells)

69. A 52-year-old man presents to you in clinic as a new patient. It has been several years since he has seen a physician. He comes to you today because he has not been feeling well and he thinks something is wrong. He reports that for the past several weeks, he has been experiencing malaise, subjective weight loss, and fevers. He also reports some swollen lumps on his neck. Physical examination is notable for lymphadenopathy and splenomegaly. Laboratory data reveal a moderately decreased hemoglobin level, thrombocytopenia, and a moderate leukocytosis. You suspect acute lymphoblastic leukemia (ALL). Further studies confirm your suspicion. A bcr-abl fusion gene is identified.

Which of the following statements is the most accurate regarding cure of ALL?

A. A combination of vincristine, prednisone, and daunorubicin cures about one third of patients with Philadelphia positive (Ph+) ALL
B. A combination of L-asparaginase and cyclophosphamide cures about one third of patients with Ph+ ALL
C. Allogeneic stem cell transplantation cures about one third of patients with Ph+ ALL
D. There are currently no regimens that are known to cure this disease

Key Concept/Objective: To know the regimen that is associated with cure of Ph+ ALL

Ph+ ALL is identified by the t(9;22)(q34;q22) or the bcr-abl fusion gene. It is currently the major challenge in curing ALL because it makes up 25% to 30% of adult cases and perhaps one half of B-lineage ALL. Approximately 70% of patients achieve CR, but the remission durations are markedly shorter (median, 7 months) for Ph+ cases than for those without a Ph chromosome (remission of almost 3 years). As yet, no chemotherapy regimen alone appears to have the potential to cure this group of patients. In contrast, allogeneic stem cell transplantation cures about one third of patients with Ph+ ALL. The probability of relapse after transplantation is approximately 30% to 50%, further attesting to the therapy-resistant nature of this disease. The treatment for Ph+ ALL should include an intensive remission-induction chemotherapy program, followed by allogeneic stem cell transplantation in the first CR if a donor is available. Considerable interest exists in investigating new agents, especially the tyrosine kinase inhibitor imatinib mesylate, in this high-risk group of patients. (Answer: C—Allogeneic stem cell transplantation cures about one third of patients with Ph+ ALL)

70. A 50-year-old man is referred to your clinic by the blood bank for a positive HTLV-I serology. He is healthy with no symptoms. He has donated blood for a workplace blood drive.

What advice would you give this patient at this time?
A. He has a 20% lifetime risk of developing leukemia
B. He has a 40% lifetime risk of developing leukemia
C. He is at increased risk for Burkitt lymphoma
D. He is unlikely to have any medical problems associated with this virus
E. He is at risk for developing an AIDS-like illness

Key Concept/Objective: To be able to recognize that most patients exposed to the HTLV-I virus will not develop leukemia

Blood banks commonly screen donated blood for HTLV-1. This virus had been linked to acute T cell leukemia and cutaneous T cell lymphoma in adults. However, most people with antibodies to HTLV-I remain free of these associated diseases, which suggests a multifactorial process in the development of leukemia. Burkitt lymphoma is associated with EpsteinBarr virus. (Answer: D—He is unlikely to have any medical problems associated with this virus)

71. Which of the following groups has an increased incidence of acute leukemia?
A. Men
B. Persons of higher socioeconomic status
C. Adults older than 50 years
D. Whites
E. All of the above

Key Concept/Objective: To know the risk factors for acute leukemia

All of the groups listed have a higher risk of developing acute leukemia than does the general population. Other risk factors include Jewish ethnicity, prior exposure to ionizing radiation (either through environmental exposure or as part of a treatment regimen), exposure to some industrial chemicals, several chemotherapy agents, a genetic predisposition, and the presence of specific diseases such as Down syndrome. (Answer: E—All of the above)

72. Which of the following statements is more commonly associated with acute myeloid leukemia (AML) than with ALL?
A. It accounts for the majority of cases of acute leukemia in adults
B. The CNS is a relatively common site of relapse
C. Patients are more likely to have hepatosplenomegaly and lymphadenopathy at presentation
D. Maintenance chemotherapy generally lasts 1 to 3 years
E. The Philadelphia chromosome–positive (Ph+) variant is more resistant to standard treatment

Key Concept/Objective: To know the differences between AML and ALL in adults

AML accounts for about 80% of acute leukemias in adults and is most likely to present with hemorrhage or infection. Standard induction therapy with cytarabine and daunorubicin (7 + 3 regimen) is followed by consolidation chemotherapy but generally no longterm maintenance regimen. ALL typically presents with constitutional symptoms (fatigue, weight loss, night sweats), and organomegaly and lymphadenopathy are more likely to be present on exam. Because CNS involvement occurs in 5% of patients with ALL, CNS prophylaxis is a standard part of treatment, as is maintenance chemotherapy. Ph+ ALL is less responsive to standard chemotherapy regimens. (Answer: A—It accounts for the majority of cases of acute leukemia in adults)

73. A 47-year-old man presents with gum bleeding, rectal bleeding, and fatigue. He is found to have disseminated intravascular coagulation (DIC). His white blood cell count is 27,000.

Which of the following conditions would best fit with this clinical presentation?
A. Burkitt cell ALL
B. Acute promyelocytic leukemia (M3)
C. Myelodysplastic syndrome
D. AML in patients older than 65 years
E. Ph+ ALL

Key Concept/Objective: To recognize acute promyelocytic leukemia as a distinct disease with a significant associated complication

DIC is frequently found at presentation or soon after induction of chemotherapy in patients with acute promyelocytic leukemia (FAB M3). Hemorrhage secondary to DIC is responsible for a high pretreatment or early-treatment mortality. DIC in this setting has been treated with heparin. Acute promyelocytic leukemia is unique in its response to all trans-retinoic acid, which is used alone or in combination with more standard regimens for induction. (Answer: B—Acute promyelocytic leukemia [M3])

For more information, see Larson RA: 12 Oncology: XVI Acute Leukemia. ACP Medicine Online (www.acpmedicine.com). Dale DC, Federman DD, Eds. WebMD Inc., New York, July 2003

Chronic Myelogenous Leukemia and Other
Myeloproliferative Disorders

74. A 55-year-old man presents to your clinic with complaints of generalized fatigue, weight loss, and abdominal discomfort with early satiety. On physical examination, the patient is afebrile and appears thin. His abdominal examination is notable for massive splenomegaly. No adenopathy is identified, and the liver is of normal size. A complete blood count (CBC) reveals a neutrophilic leukocytosis, and you suspect chronic myelogenous leukemia (CML).

Which of the following statements regarding CML is false?
A. CML is a myeloproliferative disorder (MPD) and represents a clonal disorder of the pluripotential hematopoietic stem cell
B. The CBC often reveals thrombocytosis, neutrophilic leukocytosis, and basophilia
C. The presence of the Philadelphia chromosome (Ph) is characteristic of
CML and is a poor prognostic sign
D. The three main phases of CML are the chronic phase, the accelerated phase, and the blast phase

Key Concept/Objective: To understand the pathogenesis and clinical course of CML

MPDs represent clonal disorders of the pluripotential hematopoietic stem cell and include CML, polycythemia vera, essential thrombocythemia, myeloid metaplasia, and idiopathic myelofibrosis. CML accounts for 15% of all cases of leukemia in adults. Males are affected more often than females, and the median age at presentation is 45 to 55 years. CML is caused by the transforming capability of the protein products resulting from the Ph translocation t(9;22). Up to 95% of patients with CML express Ph, which results from a reciprocal translocation between the long arms of chromosomes 9 and 22. Patients with CML who do not have Ph translocation have a significantly worse prognosis than do patients who test positive for the bcr-abl gene. CML is characterized by expansion of myeloid progenitor cells at various stages of their maturation, by the premature release of these cells into the circulation, and by their tendency to home to extramedullary sites. Symptoms at presentation reflect the increase in mass and turnover of the leukemic cells. Patients may complain of lethargy and weakness, night sweats, and weight loss. Occasionally, the spleen enlarges, causing an increase in abdominal girth and abdominal discomfort. The prognosis for patients with CML has changed significantly in the past 2 decades. Patients who are diagnosed with chronic-phase CML can expect a median survival of 5 to 7 years. (Answer: C—The presence of the Philadelphia chromosome [Ph] is characteristic of CML and is a poor prognostic sign)

75. A 63-year-old woman presents to your clinic with a complaint of increasing abdominal girth; hepatosplenomegaly is detected on examination. CBC reveals a hematocrit of 52% and a platelet count of 900,000 cells/mm3. You suspect that she has polycythemia vera (PV).

Which of the following statements about PV is true?
A. The increase in red blood cell (RBC) mass is mainly the result of an increase in the level of erythropoietin
B. Thrombotic complications are rare in PV
C. To diagnose PV, independent determination of RBC mass and plasma volume by isotope dilution is mandatory
D. The standard of care for treatment of PV is an aggressive chemotherapy regimen

Key Concept/Objective: To understand the clinical presentation, diagnosis, and treatment of PV

PV is a clonal disorder of hematopoietic stem cells. Unlike in CML, however, no clear causative cytogenetic-molecular lesion has been identified. Expansion of RBC mass is caused by increased production by hypercellular bone marrow and is not dependent on serum levels of erythropoietin. Indeed, erythropoietin levels are typically low in patients with PV; this distinguishes PV from the secondary polycythemia associated with certain tumors (e.g., renal cell carcinoma and hepatocellular carcinoma) and with pulmonary, cardiac, and renal disorders. The clinical manifestations of PV are a consequence of the excessive proliferation of hematopoietic cell lines and are mainly characterized by microvascular and macrovascular thrombotic events. On physical examination, the most common findings in patients with PV are ruddy cyanosis, hepatosplenomegaly, conjunctival plethora, and hypertension. Unlike essential thrombocytopenia, venous and arterial thromboses occur with equal frequency in PV. Microvascular symptoms such as acroparesthesias, erythromelalgia, peripheral gangrene, and ischemic neurologic and visual disturbances are frequent. Particularly serious thrombotic events involve cerebral and coronary vessels; hepatic veins; the inferior vena cava (Budd-Chiari syndrome); and mesenteric vessels. The diagnosis of PV requires exclusion of secondary causes of increased red blood cell mass and blood volume. Independent determination of RBC mass and plasma volume by isotope dilution is mandatory. Phlebotomy, low-dose aspirin, and possibly hydroxyurea represent the best approach to treatment. (Answer: C—To diagnose PV, independent determination of RBC mass and plasma volume by isotope dilution is mandatory)

76. A 45-year-old man presents to your clinic with painful, unilateral swelling of the right lower extremity; he has been experiencing these symptoms for 48 hours. The patient has no known predisposing factors for deep vein thrombosis (DVT) and no family history of DVT. Ultrasonography reveals a femoral vein DVT. A CBC is obtained; the patient’s platelet count is 1,200,000 cells/mm3.

Which of the following statements regarding essential thrombocytosis (ET) is false?
A. In patients with ET, an elevation in the platelet count is caused by increased production by megakaryocytes in conjunction with normal platelet survival
B. To diagnose ET, you must exclude iron deficiency, malignancies, inflammatory conditions, and infections
C. The platelets in patients with ET are functional, and hemorrhage rarely occurs
D. Unlike PV, ET rarely progresses; leukemia develops in only 3% to 4% of patients

Key Concept/Objective: To understand the clinical presentation, diagnosis, and treatment of ET

The elevated platelet count in patients with ET is caused by increased production by megakaryocytes in conjunction with normal platelet survival. ET is diagnosed after secondary causes of elevated platelet counts (e.g., iron deficiency, malignancies, inflammatory conditions, and infections) have been excluded. Major thrombotic complications occur in 20% to 30% of patients. Thrombotic complications frequently manifest as DVT and pulmonary embolism. Thrombosis of hepatic veins leads to Budd-Chiari syndrome; thrombosis of renal veins can cause nephrotic syndrome. Erythromelalgia and digital ischemia constitute microvascular forms of arterial thrombosis in ET. Hemorrhagic events occur in up to 40% of patients, with the gastrointestinal tract, urinary tract, skin, eyes, and brain being possible bleeding sites. Individual patients can suffer from both thrombotic and hemorrhagic episodes. Unlike PV, ET rarely progresses, and leukemia develops in only 3% to 4% of patients. The correlation between the degree of thrombocytosis and the risk of thrombosis is poor. Untreated asymptomatic patients and those who are at low risk for thrombohemorrhagic complications (i.e., those with no history of thrombohemorrhagic episodes, those younger than 60 years, and those whose thrombocytosis is of shorter duration) may have the same life expectancy as an age-matched and sex-matched control group without ET. The indications for therapeutic intervention have to be considered carefully with regard to the risk of vascular complications. In general, treatment should be considered only in patients at high risk for thrombohemorrhagic events. (Answer: C—The platelets in patients with ET are functional, and hemorrhage rarely occurs)

77. A 52-year-old man is seen for annual examination. He reports increasing fatigue over the past 3 months, accompanied by a 10 lb weight loss and a sense of abdominal fullness. On examination, he is slightly pale, with dullness to percussion in the left upper quadrant of his abdomen. The remainder of his exam is normal, with no bruising or lymphadenopathy. A CBC reveals a WBC count of 117,000, predominantly neutrophils in all stages of maturation.

Which of the following other findings would you expect to see in this patient upon further evaluation?
A. Basophilia on peripheral smear
B. A BCR-ABL translocation
C. A Philadelphia chromosome
D. A decreased leukocyte alkaline phosphatase (LAP) score
E. All of the above

Key Concept/Objective: To know the common laboratory findings in a patient with chronic myelogenous leukemia (CML)

CML is an acquired clonal stem cell disorder in which more than 90% of patients express the Philadelphia chromosome (a translocation between chromosomes 9 and 22) on cytogenetic analysis. Genetic material is exchanged between the BCR and ABL genes. Other common laboratory findings include a WBC count over 100,000, with a neutrophilic predominance, anemia, thrombocytosis, basophilia, and a decreased LAP score. (Answer: E— All of the above)

78. An active, otherwise healthy 32-year-old man with type 1 diabetes is found to have a hematocrit of 60%, a platelet count of 556,000, and a WBC count of 8,050 on routine blood work. These values were confirmed on three separate occasions. His examination is unremarkable except for a mildly elevated blood pressure, at 148/92 mm Hg. He has no hepatosplenomegaly or lymphadenopathy.

Which of the following would you expect to find in this patient on further evaluation?
A. An elevated erythropoietin level
B. Splenomegaly on abdominal ultrasound
C. An elevated transferrin saturation
D. An oxygen saturation of < 92%
E. Cytogenetic abnormalities

Key Concept/Objective: To know the confirmatory tests and their expected results in a patient with polycythemia vera

Despite this patient’s young age, polycythemia vera would best explain his laboratory values, lack of symptoms, and examination findings. The next step is to exclude other causes of an elevated RBC mass (pulmonary, renal, and hepatic disease), as well as to confirm the presence of polycythemia vera. This young man with diabetes should be screened for hemochromatosis, but hemochromatosis would not explain his constellation of hematologic findings. A normal oxygen saturation (expected in this healthy, asymptomatic man) and splenomegaly are two of the major diagnostic criteria for polycythemia vera. A normal clinical examination does not preclude the presence of splenomegaly, and an ultrasound should be ordered. Erythropoietin levels are low in polycythemia vera, differentiating it from many of the secondary causes of elevated RBC mass, and cytogenetic abnormalities are found in only 20% of polycythemia vera patients. Cytogenetic analysis is generally pursued because the presence of an abnormality confirms the diagnosis and can have prognostic and treatment implications. (Answer: B—Splenomegaly on abdominal ultrasound)

79. The patient in Question 78 is started on twice-weekly phlebotomy, and his hematocrit stabilizes at 39%.

Which of the following statements regarding this patient’s prognosis is true?
A. His life expectancy will be unchanged as long as he maintains a normal hematocrit
B. Addition of hydroxyurea to his treatment regimen could cure this disease
C. Phlebotomy will reduce his risk for thrombotic complication
D. His median expected survival time is 15 to 20 years
E. He has a 50% chance of developing leukemia by the age of 40 years

Key Concept/Objective: To know the common complications and prognostic factors in patients with polycythemia vera

The major complications of polycythemia vera are venous and arterial thrombosis, myelofibrosis (“spent phase”), and acute leukemia. Half of patients develop leukemia within 20 years of diagnosis. Although treatment can help reduce the risk of leukemic transformation and lower the rate of thrombosis, life expectancy will be significantly reduced in this young man. Low-dose aspirin and reduction of significantly elevated platelet counts with agents such as hydroxyurea are effective in controlling vascular complications. Phlebotomy does little to reduce the thrombotic risk. (Answer: D—His median expected survival time is 15 to 20 years)

For more information, see Faderi S, Kantarjian HM: 12 Oncology: XVII Chronic Myelogenous Leukemia and Other Myeloproliferative Disorders. ACP Medicine Online (www.acpmedicine.com). Dale DC, Federman DD, Eds. WebMD Inc., New York, April 2004

Head and Neck Cancer

80. A 37-year-old native Alaskan man presents with left-side otalgia of 2 months’ duration. He has difficulty breathing through the left side of his nose. A nasopharyngeal mass is discovered; the mass is identified histologically as a lymphoepithelioma. He does not use cigarettes or alcohol.

Which of the following is true for this patient?
A. He should be given a course of trimethoprim-sulfamethoxazole
B. He should have a chest CT scan
C. Considering his age and smoking history, it is unlikely that this is a malignancy
D. This may be a malignancy linked to a viral infection
E. This is probably a malignant lymphoid cell tumor

Key Concept/Objective: To know the distinction between endemic and sporadic nasopharyngeal carcinoma

Nasopharyngeal carcinoma occurs in two distinct forms. The most common is sporadic squamous cell carcinoma, usually seen in older patients who have a long history of smoking. A second endemic form is seen in Native Americans living in Alaska, Mediterraneans, and Southeast Asians. It is linked to Epstein-Barr virus, and it is likely that other risk factors are required as well. The lymphoid cells are normal T cells that infiltrate the epidermoid tumor. Antibiotics are not indicated in this case, nor is a chest CT scan, because this is neither a lymphoma nor a vasculitic or granulomatous process and therefore distant metastases are unlikely. (Answer: D—This may be a malignancy linked to a viral infection)

81. A 71-year-old man with a long history of tobacco and excessive alcohol use is found to have a 3 cm firm right anterior cervical lymph node. Fine-needle aspiration of the node reveals squamous cell carcinoma. He has no symptoms or obvious lesion to suggest the primary site.

What should be the next step in caring for this patient?
A. Watchful waiting
B. Bone scan
C. Initiation of treatment with node excision followed by radiation therapy
D. Induction chemotherapy with cisplatin and fluorouracil
E. Panendoscopy under anesthesia with biopsy of sites at which primary head and neck cancers frequently occur

Key Concept/Objective: To understand the diagnosis of head and neck cancer with unknown primary site

The most common sites of cancers in this circumstance are the base of the tongue, the nasopharynx, and the piriform sinus. If no primary site can be identified, biopsies should be performed at these sites as part of the diagnostic and staging evaluation. If no primary site is discovered after these biopsies, treatment is based on the nodal stage. The goals of treatment for locoregional disease are cure and preservation of function. Chemotherapy may indeed be part of the initial treatment modality, but it should occur only after attempts are made to identify the primary site by biopsies performed under anesthesia. (Answer: E—Panendoscopy under anesthesia with biopsy of sites at which primary head and neck cancers frequently occur)

82. Which of the following statements about the treatment of head and neck cancer is true?
A. Cure is unlikely, even with early-stage disease
B. Development of a second primary tumor after successful curative treatment of early-stage disease is rare
C. Concomitant chemoradiotherapy has resulted in increased disease-free intervals and in some studies has increased survival
D. Radical surgery is reserved for patients with recurrent disease
E. Induction chemotherapy for locoregional disease has resulted in tumor shrinkage and preservation of the larynx as well as increased overall survival

Key Concept/Objective: To understand the treatment of head and neck cancer

Concomitant chemoradiotherapy involves sensitizing tumor cells to radiation by administering chemotherapy, usually cisplatin and fluorouracil, during radiation therapy. Use of concomitant chemoradiotherapy has led to improvements in the control of locoregional disease, with some studies suggesting an increase in the 3-year survival rate from 30% to 50%. Other studies have shown increased overall survival with concomitant chemoradiotherapy. This therapy may also lead to improvements in the preservation of organ function in patients who require less surgery. Concomitant chemoradiotherapy is now considered standard care for a majority of patients with locoregional disease. Early-stage disease is treated initially with either surgery or radiation therapy, depending on tumor location; this therapy results in a 60% to 90% cure rate. However, the risk of developing a second head and neck cancer is 3% to 5% per year. In recurrent or metastatic disease, chemotherapy is the standard approach for preserving quality of life while providing palliation. Induction chemotherapy leads to tumor shrinkage, laryngeal preservation, and decreased disease in areas other than the head and neck, presumably by eradicating micrometastases.

However, survival is not improved with this modality because the locoregional control achieved is not better than that achieved with surgery and radiation therapy. (Answer: C— Concomitant chemoradiotherapy has resulted in increased disease-free intervals and in some studies has increased survival)

For more information, see Vokes EE: 12 Oncology: XVIII Head and Neck Cancer. ACP Medicine Online (www.acpmedicine.com). Dale DC, Federman DD, Eds. WebMD Inc., New York, November 2000